You are on page 1of 81

Cell Colony Lab Source & Virulence Factors Associated Host Treatment /

Features Features Characteristics Transmission (disease causing) Diseases Defenses / Prevention


Immunity
Staph Aureus Gram (+) Small, 2- Non-motile Can colonize nose & - Beta lactamase pen resistant Localized Infections: No long term Methicillin
8mm - Catalase (+) skin; Not normal - Mutatnt penicillin-binding skin infections immunity
Cocci in - Coagulase (+) Flora; GI & genital proteins (methicillin - Folliculitis (plug hair) therefore Vancomycin (if
grape Yellow- - Dnase (+) tracts; Transmitted by resistance) - cellulites recurrent MRSA)
cluster golden -Mannitol (+) fomites, sneezes, food - Impetigo (vesicular infections
opaque, - COAGULASE lesion), Furuncle, possible Bacitracin
1 u in Tellurite reduction + - EXOTOXINS - Protein A (opsina. Attack Scalded Skin Syndrome (topically)
diameter Beta (black colonies), - Cytoxins alpha, beta, bacteria) (Ritter’s Disease), Toxic CD4+ T-cells
hemolytic gamma, delta toxins - Polysaccharide A epidermal necrolytic release Surgical
Colonies on Beta Hemolytic - Enterotoxins - Staph. Decomplementation TEN disease cytokines debridement
blood agar, - toxin A (foodpoison) on antigen Systemic Infections:
facultative Growth, presence of - toxin B, entercolitis - Var Enzym (Spread Fact) Food poisoning, TTSS, Opsonization Drainage of
anaerobic, 7.5% NaCl (superinf) - lipase, protease, Osteomyelitis, Infective by IgG, etc wound
37C - emesis (vomit) hyaluronidase, Nucldease, (septic) Arthritis, Acute
-Sens.Novabiocin - Toxic ShockS Toxin DNase (staphylodornase), Bacterial Endocarditis,
- exfoliatin staphylokinase (break fibrin) Post-viral Lobar
Pneumonia (empyema),
Bactermia and Sepsis
Staph. Gram (+) Small, 2-8 - Catalase (+) - Colonize human - No Vancomycin
Epidermis mm - Coagulase (-) skin & mucus memb Betalactamase (pen resistant) Systemic Infection: immunity
Cocci in - Dnase (-) - Normal Flora of the Mutant penicillin-binding - Bacteremia & sepsis against (organism is
cluster White - Mannitol (-) skin proteins (methicillin - Subacute bacterial previous resistant to most
(opaque), - May cause disease in resistance) same as S. endocarditis infection others)
1 u in Round, - Gamma immunocompromised Viridans
diameter Hemolytic patients following - Intact skin is
Gamma trauma,also iatrogenic Exopolysaccharide important
hemolytic Growth in presence intro, or by IV needles glycocalyx defense
colonies on of 7.5% NaCl (SLIME LAYER) – sticks to - CD4 + T-
blood agar Spreads by heart Cells release
(No -Sens.Novabiocin hematogenous route cytokines
hemolysis) Urease (+) Multiple drug resistance
Opsonizat
IgG,
Staph. Gram (+) Small, 2- - Catalase (+) Colonizes human Penicillin resistance Urinary Tract Infect. No immunity Trimethoprim-
Saprophyticus 8mm; white - Coagulase (-) genitourinary, skin Hemagglutinins & other cell Upper UTI = against Sulfamethoxazole
Cocci in (opaque), - Dnase (-) (urogenitory) tract, surface proteins may mediate pyelonephritis previous (bactrim)
cluster but older - Mannitol Var infection
colonies Depending on Mucous membranes Attachment to epithelial Lower UTI = cystitis
1 u in maybe strain, Growth in (lesser extent, GI) cells CD4+ T-Cells
diameter yellowish, presence of 7.5% Pyuria (puss in urine) release
round NaCl Inf. due to poor Urease: may mediate host cytokines
- Resistant to hygiene & sexual pathogenesis
Gamma Novabiocin, activity, especially in Opsinization
hemolytic - Urease (+) young females by IgG
Cell Colony Lab Source & Virulence Factors Associated Diseases Host Treatment
Features Features Characteristics Transmission Defenses & /
Immunity Prevention
Strep Gram (+) Small, gray- Non-motile Human skin * Hyaluronic acid * Skin infection Intact skin is Active
Pyogenes white, round * Impetigo (streptococcal defense infection use
Cocci in Catalase (-) Mucous * Hemolysins (SLO & SLS) pyoderma) Penicillin G
Group A chains Beta membranes of - Streptolysins O Cross-reactive or
hemolytic Dnase (+) oropharynx (5- * beta hemolysis RBC, 02 * Erysipelas : red rash on AB ==> ARF. erythromycin
1 u in 15%) & vaginal * sens. => rhematic fever AGN face (slapped cheek) Ag-ab
diameter Beta Hemolytic tract - Streptolysins S complexes ==> Bacitracin
* stable w/ O2, little to cause * Necrotizing fasciitis: flesh AGN. Look cream (GAS
No growth > 6.5% Transmitted by disease eating strep disease. presence of susceptible,
NaCl respiratory SPREADS! antibody to other Strep
droplets from * Erythrogenic toxin (3) causes Streptolysin O are not)
Sensitive contaminated food red rash, Scarlet fever (use Dick Systemic Infections and
bacitracin Test) chks toxin level, if red rash, Pharyngitis (strep throat, streptodornase
no AB) fever producing tonsillitis)
(pyrogenic) are superantigens
(strep. Pyrogenic exotoxin) Scarlet fever (scarlatina)
infection w/ erythrogenic
* M Protein : antiphagocytic leads to red “sandpaper”
- > 100 different serotypes rash on trunk or strawberry
Essential for pathogenicity tongue
- M proteins resemble x-reactive
antibodies lead to rheumatic Puerperal fever (child birth
fever fever) uterine infection
- antiphagocytic
- adhesion factor Poststreptococcal infection
sequelae
* Hyaluronidase degrades - Acute Glomerulonephritis
hyaluronic acid (AGN): NO viable
organisms present, facial
* Streptokinase: lyse clots edema & smoky urine
(spreading factor) - Acute Rheumatic Fever
(ARF): NO viable
* Nuclease:degrade D/RNA organisms present, include
Spreading factor â pus viscosity migratory arthritis (aseptic),
DNase B = streptodornase (á M protein & heart tissue
antibody titer indicates recent S. leads to heart damage
pyogenes infection
Cell Colony Lab Source & Virulence Factors Associated Diseases Host Treatment /
Features Features Characteristics Transmission Defenses & Prevention
Immunity
Strep Gram (+) Small, gray- Non-motile Human skin Capsule: polysaccharide Neonatal meningitis (fever, IgG to capsule Ampilicillin
agalactiae white, Type III capsule is composed lethargy, & seizures) plus
Cocci in round Catalase (-) Mucous membranes of sialic acid (serum invades through respiratory PMN attack aminoglycosides
Group B pairs of vagina, male resistance) tract strep for infants
Beta CAMP test (+) urethra, throat, GI - Early onset: 0-5 days
1 u in hemolytic Beta Hemolytic tract Hemolysin infection (in utero) Penicillin G for
diameter - Late onset: 5-90 days adults (*
No growth > 6.5% Infects newborns pregnant to
NaCl during birth Neonatal pneumonia in carry GBS)
(Puerperal fever) utero infection
Resistant Vancomycin for
bacitracin Post-partum endometriosis Penicillin
(puerperal fever) sensitive
Hydralize sodium patients
hippurate Immunocompromised leads
to pneumonia, septicemia, Vaccine, Pen. G
prosthetic disease, develops
puerperal sepsis, skin protective IgG
infection cross placenta &
protect fetus
Urinary tract infections
Cell Colony Lab Source & Virulence Factors Associated Diseases Host Treatment /
Features Features Characteristics Transmission Defenses & Prevention
Immunity
- Strep bovis Gram (+) Small, gray- Non-motile Enterococci colonize S. Bovis none (killed easily) Bacteremia: S. bovis Ab important Penicillin G for
white, at the GI tract. invades blood via GI route. and PMN can S. bovis,
- Enterococcus Pairs and round Catalase (-) Commensals at the E. faecalis: high resistant to attack Strep. vancomycin
faecalis chains genitourinary tract antibiotics (vancomycin E. faecalis becomes blood- for Pen.
variable variable resistance) Adheres to borne via urinary tract Sensitive
Group D 1 u in hemolytic hemolysis Infection ==> blood damaged heart valves & infection (UTI) or GI patiens.
diameter response (GI to GU tracts) urinary tract epithelial cells. route.
bile-esculin E. faecalis: use
(black ppt) S. Bovis found in GI UTI enterococal inf. Leads combination of
tract to cystitis (lower) or vancomycin &
pyelonephritis (upper) aminoglycoside
Enterococcus
grows at 6.5% Sub-acute endocarditis:
NaCl (+, present) from bacteremia.
Enterococcus ONLY infects
Group D, S Bovis abnormal valves or
Strep are (-, not prostheses.
present)
Bilary tract disease &
Does not grow in intraabdominal abscesses
manitol!
Colon cancer prone to S.
bovis
Cell Colony Lab Source & Virulence Factors Associated Diseases Host Treatment /
Features Features Characteristics Transmission Defenses & Prevention
Immunity
Strep Gram (+) Small, gray- Non-motile Colonizes Capsule (Specific Soluble Lobar Pneumonia *: in Antibodies Multivalent, 23
pneumoniae white, round human upper substance) 80 serotypes adults & sickle cell against capsule most common
Pairs and Catalase (-) respiratory tract Identified w/ Capsule anemia patients. offer type- capsular ag
(pneumococcus chains alpha (commensal) “swelling” Quellung rxn Symptoms: specific vaccine confers
or diplococcus) (diplococci) hemolytic alpha hemolytic* - fever, productive cough, resistance immunity for a
also part of S. (via pneumolysin People Capsule protects phagocytosis dull chest percussion, X- few years
mitis Lancet Encapsulated activity) susceptible to Non-encap. strains avirulent. ray dxn. Maybe fatal Pneumolysin
shaped colonies viral infection, Diagnosis: in high Penicillin G
(No Group b/c (smooth) (+) for bile allergy C reactive protein (CRP) and - Large # of strep & PMN concentration (sulfonamides)
lack “C” cell 1 u in solubility (10% Na malnutrition, used as indicator inflam in sputm. Often sequela leads to
wall antigens diameter Non- desoxycholate) alcoholism, response as a marker to viral infection, activation of Vancomycin
NO Lancefield encpasulated debilitation, â alcoholism, & smoking complement for penicillin
group) Grows better are (rough) Sensitive for ciliary motion F antigen: part of “C” (all disrupt fxn of cilia) sensitive indv,
in á C02 optochin (ethyl ==> aspirate into carbohydrate, hide capsule but
(capnophilic) hydrocupreine) lungs Meningitis *: in adults. vancomycin
induce lysis of S. Ag are only exposed in non- Results from bacterimia, NOT effective
Encapsulated pneumoniae Sickle cell encapsulated pneumococci sinusitis, or otitis media, for meningitis
cell anemia disease skull fracture, other b/c doesn’t
(+) for inulin Pneumolysin similar to injury. penetrate the
Autolysis fermentation Age related Streptolysin O, Listeriolysin Symptoms: BBB
(amidase O, & Tetanolysin (found in S. - fever, stiff neck, (vancomysin
activity) Inf. Indicate Spread by Pyogenes, Listeria, C. Tetani) headache, maybe fatal doesn’t work
excess # of alpha droplet nuclei fxn to inhibit ciliary movemt, Diagnosis: w/ meninges)
Naturally hemolytic â bactericidal actv PMN, -lumbar puncture,culture
competent colonies on blood inhibit lymphocytic prolif.
agar plates w/ Sinusitis *: Often sequela
neomycin. Neuraminadase remove sugar of allergy, or viral infect
from host glycoprot prevents drainage
Quellung
Neufield Rxn Peptide permeases: á Otitis media *: Often
(swelling) adherence to host tissue sequela of allergy, viral
infection prevents
IgA protease: cleaves Iga Eustachian clearance
(blocks opsonization surface) Bacteremia (30% in
- Adhesins: fibronectin pneumonias) & 80% in
- Autolysin: release toxin, infl Menigitis
- Naturally competent *common causative agent
alpha hemolytic: partial breakdown of RBC, * can be beta hemolytic if grown anaerobically but usually considered as alpha hemolytic
Test S. Pneumonoccus Viridans Strep.
Bile soluble Cell lyses Not sensitive (resistant)
Optochin Cell lyses Not sensitive (resistant)
Cell Colony Lab Source & Virulence Factors Associated Diseases Host Treatment /
Features Features Characteristics Transmission Defenses & Prevention
Immunity
Viridans Gram (+) Small, gray- Non-motile Organisms are Dextran Glycocalyc Subacute bacterial Bacteria killed Penicillin G
Streptococci white, round normal flora of (exopolysaccharide): endocarditis common by host and
Pairs or Catalase (-) does oropharynx adherence to defective heart causative agent, from immune system aminoglycoside
Group chains alpha not produce (commensal) valves, block penetration of dental work & (antibodies & before dental
includes S. hemolytic peroxide antibiotics. Forms via bacteremia. Infects heart complement) surgery w/
Mutans, S. 1 u in Infection caused GLUCOSE metabolism valves & prosthetic patients w/
Mitis, S. diameter alpha hemolytic by bacteremia devices Patients w/ heart
Salivarius, S. following dental Lipoteichoic acid: mediates heart conditions.
Sanguis (-) for bile work. Can be adhesion to fibronectin in Dental caries (tooth conditions are Vancomycin
solubility & present in GU blood clots on defective heart decay) due to S. Mutans at risk of for patients
resistant to tract and can valves which are localized by complications allergic to
(No Group b/c optochin lead to UTI large amounts of acid by penicillin.
lack “C” cell Glucan polysaccharide : fermentation of sugars in
wall antigens (-) for inulin produced by S. Mutans from the mouth. Flourine in
NO Lancefield fermentation SUCROSE in mouth thereby water prevents adhesion.
group) allowing attachment of
No group bacteria to tooth enamel.
carbohydrate
Acids (lactic acid)
Media grown on
sugar produce
glycocalyx
composed of
dextran

Normal & Damaged Abnormal & Damaged


Acute Endocarditis Subacute Endocarditis
- S. Aureus - S. Epidermis
- S. Bovis
- Enterococcus faecalis
- viridans
Cell Colony Lab Source & Virulence Factors Associated Host Treatment /
Features Features Characteristics Transmission Diseases Defenses & Prevention
Immunity
Bacillus Gram (+) Large, gray- Catalase (+) Organism Capsule: POLYPEPTIDE Cutaneous Anthrax: Ab against Vaccine for
Anthracis white, flat, (spores) composed of D- round necrotic black polypeptide humans confers
Rod, boxcar waxy, erose Non-hemolytic, normally found GLUTAMATE (NOT ulcer (ESCHAR) on capsule. short-lived
shaped colonies on NON-MOTILE in soil. polysaccharide!) skin, painless. immunity (require
blood agar can stain Ab to PA annual boosting)
Pairs or polypeptide Reservoir for Capsule is anti-phagocytic Toxin is released & prevent composed of
Bamboo chains medusa head capsule w/ spores are blocks capillaries, if binding of LF extract from a
fishing rod colonies methylene blue herbivores, Anthrax toxin: plasmid untreated leads to and EF to PA. virulent but NON-
appearance NON- rough w/ contaminated encoded, comprised of 3 bacteremia (grows in If no ab, no ENCAPSULATED
MOTILE irregular animal hides & different peptides blood stream) & cause time for anthracis
edges dust-laden, - Edema factor (EF), impairs blood to thicken. Can immune
Aerobic spore-laden phagocytic ability (swelling) be fatal if untreated in 4 response. Antibiotics:
NON- articles. - Lethal factor (LF), days. Replicates in - penicillin G
Endospores HEMOLYTIC Transmission pulmonary edema is blood! - sulfonamides
(ex. from occurs when cytolytic for macrophages. (sensitive patients)
soil) Capsule Stimulate production of Septicemia: B. - erythromycin
host contacts
induced in infected animals cytokines Skin turns black! Anthracis septicemia is - ciprofloxacin &
Spores presence of Zn metalloprotease, á rare in humans doxycycline
/ products,
resistant to 5% CO2 ==> inhales or eats cytokines (PROMPT
heat, colonies Respiratory anthrax treatment!)
spores. Spores
chemicals, appear moist - Protective Antigen (PA) (Woolsorter’s Disease)
can also enter
dryness, UV, Cellular uptake processed by pneumonia following Immunize all
via skin wounds
spore coat host protease to bind to EF or inhalation of spores uninfected animal
or abrasion.
contains LF ==> allows the factor EF from infected wool, herds!
dipocolinic or LF to be internalized by rare, but fatal. Has a
ZOONOTIC
acid & Ca++, infection host cell by endocytosis. PA latent period > 2
low water is antigenic months. Can hide in
associated w/
content cattle lung macrophages
LF and EF need PA to be
10 x 3 u in toxic GI anthrax: ingest of
diameter meat contaminated w/
B. anthracis has PA + LF or
PA +EF is NOT as virulent as spores. Mortality
EF + LF + PA. Leads to ~100%
vascular permeability &
neurotoxicity. No PA, then
â virulent, no damage.

Endospore formation:
ensures survival of
“bacteria” in harsh
conditions. Spore
germination requires O2
Cell Colony Lab Source & Virulence Factors Associated Diseases Host Treatment /
Features Features Characteristics Transmission Defenses & Prevention
Immunity
Bacillus Cereus Large Large MOTILE Vegetative cells Two Enterotoxins: CHINESE Unknown Fluid &
Gram (+) granular & spores - Necrotic toxin: heat-labile RESTAURANT electrolyte
rods in pairs colonies on AEROBIC normally found toxin (LT) that stimulates cell’s syndrome replacement if
or chains nutrient agar in soil, dust, adenylate cyclase. Causes necessary. No
Biotyping not decaying diarrhea Emetic Food Poisoning: other
MOTILE Mesophilic & used organic matter – upper GI disturbance w/ medication.
nutritionally also in rice, - Heat-stable enterotoxin (ST) vomiting toxin forms in
Aerobic require meat products acts by diff. mechanism than rice ==> consume Vancomycin
amino acid “LT”, leads to vomiting but reheated rice w/ toxin for eye
Endospores supplements Non-invase inf. not to diarrhea ==> food poisoning infections.
centrally Occurs when symptoms arise in about Multiple drug
located bacteria and/or Endospore formation: 6 hrs after eating toxin- resistance
(metacentric) spores ingested. surivival in harsh conditions. laden food.
Germination requires oxygen
Spores survive Diarrheal Food
cooking & Lecithinase (phospholipase C) Poisoning: lower GI
germinate ==> enzymes – active on cell disturbance w/ diarrhea.
produce toxin & membranes (associated w/ Toxin forms in meat /
ingest pre- ocular infection) vegetables ==> consume
formed toxin reheated foods laden w/
(food bacteria (& toxin)
Cereolysin (potent hemolysin)
intoxication) symptoms arise w/in
& Necrotic toxin (associated
w/ ocular infection) 24hrs, food inf. May last
for 2 days

Post-traumatic
endophthalmitis (eye
infection) leads to
edema. Drug abusers are
á risk, may cause
blindness

IV catheter & CNS shunt


inf & endocarditis w/
drug abusers, also
pneumonitis, bacteremia,
meningitis in
immunocompromised
Cell Colony Lab Source & Virulence Factors Associated Host Treatment /
Features Features Characteristics Transmission Diseases Defenses & Prevention
Immunity
Coryne- Slender Pleomorphic on Loeffler’s Humans are Diphtheria toxin: strong, Respiratory diphtheria Peptide B may Vaccine consist
bacterium SERUM- coagulated blood only reservoir slow acting binary toxin (has Non-invasive therefore develop in of formalin-
Gram (+) TELLURITE serum medium, for C. A:B subunit motif) has non-systemic (localized) patient, but inactivated
Diphtheriae Agar (black- stain w/ diphtheriae. tissue specificity (heart, infection => intoxicat development (TOXOID) is
Club shaped gray colonies methylene blue nerves, kidney) – acts ==> pseudomembrane maybe slow given as part of
pleomorphic indicate or toluidine blue They maybe intracellularly following in throat due to inflame. DPT vaccine to
rods tellurite to look for considered internalization by pinocytosis Response. DT toxin too children.
reduction), the metachromatic “normal” skin & Detects for cardiotoxin. small in
1.6 u long x 0.5 differences are granules, upper Pseudomembrane can amount to be Booster to
u wide usually NOT correlated respiratory tract Peptide B (shuttle protein) occur regardless of antigenic in confer long-
in L or V w/ Catalase (+) flora binds to receptor on host whether strain is natural term protection
shaped pathogenicity cells & aids Peptide A in its toxigenic or not infection “10 years”
Tellurite Horizontal transport into cell
Chinese letter C. Diphtheriae reduction transmission Intoxication: systemic Serum sickness
clumps - gravis: large occurs by Peptide A (ADP- effects of toxin ==> heart used to test
(palisades) black colonies Iron requirement respiratory riboslyating enzyme) toxicity ==> myocarditis, hypersensitivity
- mitis: small droplets or via ribosylates EF2 & stops arrhythmias, kidney , too many
Cells contain black colonies In vitro test is contaminated protein synthesis in host damage, neurological doses
metachromatic - intermedius: ELEK test, test for skin lesions in cells toxicity
phosphate- large gray TOXIN ex. cutaneous form Penicillin G
containing, colonies CardioToxin Diagnosis must be fast used to kill
Toxin encoded for by
inclusion lysogenic phage (beta and is based on clinical organism.
body for C. Diphtheriae (+) test indicates symptoms. Toxin-cell Erythromycin is
phage). Synthesis
energy storage is very ab-ag ppt toxin interaction irreversible a substitute
controlled by iron levels in
VOLUTIN sensitive to (form precipitin & antibodies are
environment; toxin
granules sunlight, soaps, line as in double expressed when iron levels ineffective once toxin is Diphtheroids
which are lost desiccation, & diffusion / low and repressed when bound refers to all
when cultured antibiotics Ouchterlongy iron levels high (repressor NON-
in vitro. assays) contains iron, no repressor Start to see endocarditis pathogenic
when no iron) When iron â w/ non-toxigenic strains Corynebacteria.
Non-motile, In vivo test use toxin expressed b/c lyse host Facultative
guinea pig skin cell Cutaneous diphtheria anaerobes.
aerobic test ==> (-) then infects open wounds They are
no redness Mycolic acid: (CHIEF Gray pseudomembrane opportunistic
VIRULENCE FACTOR!) on non-healing wounds
similar to cord factor of M. Elek test to diff.
tuberculosis, toxic glycolipid Schick skin test test for diphtheroids
protective antibodies from toxin-
K antigen: adhesion in throat (-) test: when diluate producing C.
toxin is injected , no diphtheriae
redness on skin
(+) test: redness lack Anerobic
immunity Corynebacteria
(propion-
bacterium)
Description Treatment / Prevention
Coryne-bacterium Bacteria produce porphyrins, coral red pigment Diagnosis of erythrasma is based on clinical
picture. Lesions when observed under UV
Minutissimum Organism grows on skin areas with high moisture (ex. skin folds) (365nm), Wood’s light appear pinkish to
coral red color
(Foot Associated Superificial infection w/ this organism ==> ex. Athlete’s Foot w/ scaly plaques especially
diphtheroid!) btw toes. Not pus forming and is called ERYTHRASMA Non-invasive and treated with oral
erythromycin
Cell Colony Lab Source & Virulence Factors Associated Diseases Host Treatment /
Features Features Characteristics Transmission Defenses & Prevention
Immunity
Listeria - Gram (+) Pleomorphic Catalase (+) Organism found Listeriolysin O (LLO)!!!!!!: Abortions: pregnant T Cell- Keep pregnant
monocytogenes & translucent in dust, soil, mediates escape of organism women (due to â cell mediated females away
- coccobcilli on blood B hemolysis (+) water, sewage, from phagosome. Found in S. mediated immunity) are immunity from listeriosis
or short rod agar unpasteurized Pneumonia prone to listeriosis. combats patients.
(pleomorphic) Tumbling milk, poultry, & (Spont. Abortions!) intracellular Tetracycline is
usually in Small zone motility 22C vegetables Membranolytic activity is listeria. a drug of
clumps or of beta enhanced by low pH & low Neonatal listeriosis: Listeria induces choice, or
short chains hemolysis Widespread iron (phagosome bacteria) cross in utero to fetus infected erythromycin
among animals prior to birth. Early macrophages Pen G or
Aerobic Cold storage & humans in GI Toxin is hemolytic (leads to onset: 1-2 days after to secrete IL-12 ampicillin kills
4C on blood- tract, female hemolysis) Similar to birth, most common form which organism
Tumbling containing genital tract & Streptolysin O (pneumolysin is pneumonia & sepsis promotes T
motility 22C media helps throat. & tetanolysin) Lyse vacuole ==> lead to Cells to Ampicilin &
& non-motile to enrich for membranes of macrophage, granulomatosis differentiate to gentamycin for
@ 37C Listeria Vertical monocyte and epithelial cells infantiseptica: neonatal neonatal
TH-1. TH-1
transmission: granulomas and cells produce meningitis
Facultative transplacental or Actin tail: acquires a tail abscesses of skin, eyes, IL-2 & gamma
INTRA- during birth made of actin filaments brain. Inf. Occur in utero interferon. Pasterurized
CELLULAR! directs the bacteria ==> cell maybe fatal if not treated milk
Zoonotic surface to infect Late onset: 5-90 days Gamma IFN
No spores! transmission:
neighborhood cells. post-partum, meningitis may activate
animal contact
/ meningoencephalitis listericidal fxn
or ingestion of
Ability to remove iron from similar to S. agalactiae in macrophage.
contaminated host’s transferrin
foods Immuno-compromised
Produce zinc-dependent meningitis: elderly,
Activation in phospholipase C used to lyse cancer patients, & renal
carriers who membranes (works w/ LLO) transplant patients (on
become
systemic steroid
immuno- treatments). Leading
compromised cause of meningitis
among this population.
Listeria infects
macrophages &
CSF á PMN
epithelial cells
â Glucose, cloudy
culture

Food poisoning
unpasteurized milk,
turkey
Cell Colony Lab Source & Virulence Factors Associated Diseases Host Treatment /
Features Features Characteristics Transmission Defenses & Prevention
Immunity
Clostridium Large Not usually Organism not Spores normally Botulinum toxin: MOST Botulunum food Immunity to Anti-toxin
botulinum Gram(+) rod cultured usually grown found in soil, TOXIC COMPOUNDS poisoning reinfection is needs to be
b/c dangerous. dust and in known. Eight (INTOXICATION) type specific & given early
Pairs or decaying immunologically diff. toxins permanent enough to
chains Most labs can not organic matter. produced Early symptoms: neutralize
4 groups detect toxin vomiting, nausea but no toxin monitor
type 1 Endospores Vacuum-packed Types A, B, & E are most fever. Diplopia (double hyper-
and 2 formed (sub- Spore stain useful canned goods, commonly causing human vision), dysphagia sensitivity rxn
terminal the spores may disease. (difficult swallowing), &
location) germinate & dysphonia (thickness Respiratory
under poor produce toxin. Toxins are pre-formed in speech) support
growth Food food. Labile at 121 C at 15
conditions intoxication min or boiling food for 20 Late symptoms: flaccid Surgical
min (heat sensitive) paralysis, respiratory debridement
Found in soil, * Inadequate distress ==> can be fatal of wounds &
not in canning, pH < 7, Toxins are “secreted”, released metronidazole
patients smoked fish, by cell autolysis. Infant botulism for infection
canned tuna (INFECTION): infects
Spores Botulinum toxin acts as a infant’s GI tract & Less than Guanidine HCl
resistant to Infant botulism neurotoxin. It blocks the produce toxin. Toxin1isyr old treatment
associated w/ release of acetylocholine most common source1-6 of months stimulates
boiling for
several hr honey loaded neurotransmitter from spores. Symptoms: acetylcholine
w/ spores & poisoned neurons at weakness, feeble cry, release
Obligate wound infection myoneural junctures. W/in paralysis, respiratory
anaerobic ==> subsequent 36hrs flaccid paralysis (Botox distress. Maybe fatal,
toxin poisoning for cosmetic purposes) Blocks could be cause of SIDS.
Non-motile AcH w/ infants! FLOPPY BABY Gastric
SYNDROME (limp baby lavage to
Endospores formation: syndrome) Honey may neutralize
survivial in harsh conditions contain spores! the toxin
Wound Botulism: Penicillin and
entrance of endospores trivalen
into wounds ==> vaccine
Snare protein will
undergo germinate ==> toxin
Light proteolysis
chain of 1-2
snap25,
microgram/
syntaxsin
kg)
botulium Injection of
toxim wil heroin, skin
cause popping
lysis od 18-36 hrs
snare after
protein ingestion

Inhalation
botulism,
purify con and
aerosolized-
Cell Colony Lab Source & Virulence Factors Associated Diseases Host Treatment /
Features Features Characteristics Transmission Defenses & Prevention
Immunity
Clostridium Large Not usually Toxins A & B are C. difficile is Exotoxins A and B (==> ANTIBIOTIC Commensal, no Discontinue
difficile Gram(+) rod cultured detected by lethal part of normal hemorrhagic necrosis) ASSOCIATED COLITIS strong host current antibiotic
effect on cell flora of 10% (AAC), ULCERATIVE response treatment &
Obligate cultures (within humans (á in Toxin A: enterotoxin causes COLITIS Most common substitute
anaerobic 24hrs) hospitalized) diarrhea & colitis agent; occurs in GI tract Need to restore quinolone,
(hemorrhagic necrosis). normal flora sulfonamide or
Spore- Toxin detected by 75% of neonates Cause infl. Damage to host “pseudomembranous aminoglycosides
Antibiotic formers a) Latex colonized & tissues colitis, arises few days TMP-SMZ.
associated agglutination serve as after antibiotic treatment
pseudome motile b) ELISA kit – reservoirs to (clindamycin, ampicillin Vancomycin
mbranous detect toxins A&B others in Toxin B: cytotxin lethal to & cephalosporins) best!
enterocolitis Anaerobic hospital & at cultured cells.
home Depolymerizes host actin * Explosive, bloody Metronidazole –
NEONATES are altering host cell cytoskeleton diarrhea, fever & less effective
asymptomatic ==> cell loses shape ==> pseudomembrane in
necrosis colon (detected by
Nosocomial endoscopy)
spread in adults Endospore formation
is activation in
carrier via
changed
bacterial
balance Cause disease in
associated w/
Ampicillin
US and Europe clindamycin,
antibiotic use highly virulent lincomycin,
flouroquinolo
nes
Cell Colony Lab Source & Virulence Factors Associated Diseases Host Treatment /
Features Features Characterist Transmission Defenses & Prevention
ics Immunity
Clostridium Large Double beta Obligate Spores normally 5 different serotypes based on Gas gangrene 80% Antibodies & Respiratory
perfringens Gram(+) rod hemolysis on anaerobic found in soil, dust exotoxins produced. All serotype A. Infection is other host support
blood agar & in feces serotypes ==> alpha toxin result of trauma or defenses
Anaerobic Double beta (lecithinase) surgery. Rapid spread, ineffective. Surgical
Thioglycolate hemolysis (1 Wound tissue necrosis & debridement
Sub-terminal medium ring beta / 1 contamination Exotoxins systemic intoxication Antitoxin to of wounds &
spores ring partial) with dirt leads to - Alpha toxin ==> lecithinase (via blood) alpha toxin is metronidazole
(spores in infection. = phospholipase C (converts ineffective & for infection
soil, not in Lecithinase (+) lecithin in cell membrane to Myonecrosis by alpha leads to
patients) form ppt rings C. perfringens diglyceride & toxin, gas production hypersensitivity. Refrigerate
of insoluble similar to c. novyi, phosphorylcholine) ==> RBC due to fermentation of meats after
Non-motile diglycerides c. septicum, c. lysis, destroys membranes & muscle carbohydrates No immunity to cooking
histolyticum, c. mitochondria (saccharolytic activity) reinforcement
Need to bifermentans - beta, epsilon & iota toxins Maintain
Large confirm A & C leads to necrosis & lethality Anaerobic cellulites sterility of
gram (+) serotypes Gas Large amount of (beta toxin ==> w/ pig bel) similar to gas green surgical
bacilli gangrene have C02 produced w/ limited to fascia (no instruments
different causes necrosis. Spores á Spreading factors: muscle invasion)
growth. Restricted - kappa toxin = collagenase Hyperbaric O2
blood flow ==> gas (collagen digestion & Myositos (muscle environment
gangrene / liquefication, destroy bone, infection)
myonecrosis cartilage, & skin, found in Penicillin &
extracellular matrix) Food poisoning other broad
Organism can also - mu toxin = hyaluronidase spectrum
colonize GI tract & - nu toxin = DNase Enteritis Necroticans antibiotics
female genital tract - delta toxin = hemolysin (Pig-bel) associated w/
(leads to septicemia - lambda toxin = proteinase serotype C producing Penicillin G –
& organ abscess) in (degrades gelatin & alpha & beta toxin antibiotics (pg
immuno- hemoglobin) 128)
compromised by - Enterotoxin (heat liable) Septicemia & organ
drug &/or other inhibit fluid absportion from abscesses:
disease gut associated w/ serotype A
food infection. Trypsin
Food poisioning - Neuraminidase
(beta toxin) - fibrinolysin
- theta toxin: heat & O2 labile Necrotizing
(sensitive) hemolysin; enterocolitis or pig
membranolytic bell, due to beta
toxin producing type
C

Mot: ingestion of
meat ptoducts ,
short incubation
period ( 8-24 hrs)
Cell Colony Lab Source & Virulence Factors Associated Host Treatment /
Features Features Characteristics Transmission Diseases Defenses & Prevention
Immunity
Clostridia Large Forms Culture C. tetani Spores are Exotoxins: Deep wound Ab against Vaccination with
Tetani Gram(+) rod transparent from wound site normally found a) tetanus toxin infection ==> toxin develop tetanus toxoid (@
In pairs or colonies on (anaerobic) but also in soil, dust & (tetanospasmin): coded for TETANUS. Spores in host but too 1st, formalin-
chains serum agar culture aerobically may be on plasmid, produced after enter deep wound late to stop inactivated toxin
cultured to check 2ndary introduced into germination of spores, tetanus & as part of DPT
Tennis ANAEROBICAL bacterial invaders host via released (during cell Spastic paralysis due death. with boosters give
racket or LY Staph, Strep puncture autolysis) as pretoxin & to unremitting Recovery does long lasting
drumstick (O2 sensitive) wounds, activated by bacterial muscle contraction. not confer protection)
appearance gunshots, & 3 doses
protease. immunity to 2, 34, 6
burns Diagnosis: organism reinfection DPT involves
Endospores injections
months)
- Toxin blocks the may cultured
located exocytosis of inhib. anaerobically beginning of 3
terminally Transmitters, Glycine, and months. Once
GABA. Results in spastic Difficult swallowing immunity, booster
Obligate (rigid) paralysis of voluntary due to LOCK JAW every 5-10 years.
Anaerobic muscles due to un-opposed
excitation (tetani) of motor Opisthotonos: arched Tetanus antitoxin-
Motile neurons back & neck human derived ab
to the toxin;
“LOCK JAW” (TRISMUS) Risus sardonicus respiratoryATS
for disease ==> fascial grimace support; passive
hyperbaric oxygen
Tetanolysin: is a hemolysin Respiratory distress:
Strict serologically related to Surgical
due to tetanus of
anaerobe Streptolysin O (& debridement of
non diaphragm. Resp.
listeriolysin & wounds &
encapsul arrest
pneumolysin). Also these metronidazole
ated Tetanus
are found S. Pyogenes, S. (kills bacteria)
neonatorum
Neonatal tetanus
Pneumoniiae, Listeria Lyses ==> infection of Vaccinate
RBC, PMN, macrophages, all mother Antibiotics: inhibit
umbilical stump.
bacterial growth â
platelets & fibroblasts. Infant weak on the 7
month of toxin production
Endospore formation: Localized tetanus pregnancy
C. tetani is
survives harsh conditions
sensitive to
Cephalic tetanus
Poor prognosis pencillin. Pen.
Inhibits normal
GABA.
Cell Colony Lab Source & Virulence Factors Associated Host Treatment /
Features Features Characteristics Transmission Diseases Defenses & Prevention
Immunity
Gram (+) rod Cultured Propionic acid part of normal Lipases (Lipophilic)* split Acne vulgaris PMN attracted Benzyoyl peroxide
Propionibact sometimes anaerobically production flora of skin, off fatty acids from skin (common acne) to bacterial topical
erium acnes branching mouth & eyes lipids. Inhibit other bacteria substances bacteriostatic
Indole (+) but can contribute to tissue Chronic effect by oxidation
Anaerobic grows in acne inflammation inflammatory Complement of area
(anaerobic Contaiminates lesions infection of hair activiation
diphtheroid) blood cultures, b/c Able to enhance host’s follicles. Papules, occurs Trimethoprim for
it’s on skin metabolize skin- immune system to produce comedones systemic treatment
Non-motile dervived lipids, a strong acute inflammatory (blackheads),
blocks sebaceous response pustules or cysts on Retinoids (Vit A)
Not spore! glands cuase face for cystic acne
acne & other
pustular rxn. Bacteremia: ==>
May enter via endocarditis
wounds. (opportunistic!)

Lipophilic diphtheroid (appeared in the test bank)


Cell Colony Lab Source & Virulence Factors Associated Diseases Host Treatment /
Features Features Characteristics Transmission Defenses & Prevention
Immunity
Actinomyces Gram (+) Cultured Grows on brain- Normal dental A. Israelii NOT very Actinomycosis is caused Local, acute
Israelii anaerobically heart infusion flora (in & around virulent & actinomycoses by obligate anaerobes inflammatory Prevention:
Rods agar teeth & gum are polymicrobial infections such as A. israelii. response - Amoxicillin 8-
Mold like anaerobically. margin) & vaginal involves Strep. Sanguis, 12 months
Long chain structures flora of women Strep. Mitis, Prevotella, Oral actinomycosis (no pain in - tetracycline
branch & Staining of Porphyromonas, Bacteroides, (cervical-facial): actinomycosis) - preg. Women
filamentous Colony “sulfur granules” Due to poor Fusobacterium, Eikenella, & overgrowth of normal (erythromycin)
resembles from pus, hygiene, trauma, or Treponema denticola. oral flora or soft tissue - good oral
Anaerobic molar like sputum, tissue bacterial infection. mass along the jaw. hygiene
(micro- tooth biopsy w/ Endogenous “Sulfar Granules” are
aerophilic hematoxilin-eosin activation yellow bacterial mats Chronic, suppurative, Treatment:
Lesions: (H&E) (immuno- (filaments) bound with granulamatous disease - Surgical
Non-motile forms compromised host) calcium phosphate (described as “punched debridgement
“sulfur Immuno- out” mandible bone
Produce granules” b/c fluorescence lesions) - drainage
exospores of yellow
(looks like color Aspirated into lungs: - Penicillin G
fungus but (CaPO4) leads to Lung
it’s a Actinomycosis (Farmer’s
bacteria!) Lung) chest pain,
pneumonitis, empyema,
No nucleus hemoptysis, fever, may
spread & appear in X-rays

Swallowed ==>
abdominal disease.
Infection ==> appendix,
trauma, or performated
ulcer

Pelvic Actinomycosis:
serious infection, caused
by chronic IUD
----------------------------------- (intravenous used drug)
Disease ===> section use / inflammation
----------------------------------- associated w/ STD
A = Actino
A.mycosis A.mycetoma A. bovis: “lumpy jaw”
Obligate Aerobic Actino: dental plaques
Anaerobe intracellular
Cell Colony Lab Source & Virulence Factors Associated Diseases Host Treatment /
Features Features Characteristics Transmission Defenses & Prevention
Immunity
Nocardia Gram (+) Cultured Acid fast stain of Abundant in soil & Catalase & Superoxide Nocardiosis (not Host generates Caught early:
asteroides aerobically sputum or pus for cause of Dismutase enzymes: high contagious, but lethal, & acute Sulfonamide
Short rods on blood branching, opportunistic levels resist PMN’s may relapse after inflammatory
Nocardia agar filamentous infections in intracellular oxidative burst. treatment) (pyogenic) Or else:
brasiliensis Long chains bacteria. immuno- response due to 80% fatality
Smells compromised Formation of long filaments Pulmonary abscesses: PMN
Branch & musty Use modified patients w/ AIDS, á ability resist phagocytosis cavitating lesions in lungs phagocytosis. TMP-SMZ for
filamentous acid-stain corticosteroids. ==> TB, Crhonic lobar Ab needed for propylaxis
Grows slow (sulfuric acid) . Mycolic acid pneumonia phagocytosis
Aerobic Considered Most common: (require
Rough “paritial/weak - Infection initiates Sepsis: spreads from lungs cytokine
Non-motile surface (may acid fast” via respiratory route via blood to other organs production).
have aerial (inhale) often to CNS ==> brain Activated CD8+
Stained w/ filaments) Catalase (+) abscess, skin ==> kidney T cells are toxic
modified - Enters by skin lesion to N. asteroides
(non- Superoxide trauma w/ foot the
alochol) dismutase (+)
usual site of Actinomycetoma (aerobic
acid-fast infection actinomycosis): chronic
b/c of granulomatous infection of
mycolic acid subcutaneous tissues. Sinus
6% tracts are formed draining
pus to skin surface. Can
In vivo: lead to damaged bone.
INTRA-
CELLULAR! Foot is the usual site of
infection after injury:

Nocardia asteroides Nocardia brasiliensis


pulmonary infections (from environment) and/or major cause of actinomycetoma infections of foot
actinomycetoma infections of foot
Cell Colony Lab Test Source & Virulence Factors Associated Diseases Host Treatment /
Features Features Character. Transmission Defenses Prevention
Immunity
Myco- Gram (+) Aerobically 5- Acid fast stain of Humans are the Cord factor composed of TB is a pulmonary disease, T Cell Combination of
bacterium 10% CO2 & septum, but TB reservoir trehalose dimycolate: dissemination occurs by blood mediated drugs, lasts up to
tuberculos Slender rods grows is resistant. Leads to TNF induction STEPS in disease process: delayed 12 months.
in long chains slowly!!! Horizontal transfer by immune cells, kill itself type hyper-
(Koch’s & branches Culture sputum via respiratory 1.Respiratory droplet reaches sensitivity Antibiotic resist.
bacillus) Grown w/ (egg agar) droplets from Catalase: is required for alveloi: destruct of PMN by TB (TDTH) isolates arise in
Use Acid-Fast Mueller- infected patients. isoniazid sensitivity (INH) non-compliant
stain Hinton agar, Chest X-ray: (secondary TB ex. 2. Alveolar macrophages Antibodies
Jensen- Chk calcification reactivated Protein antigens of outer ingest TB ==> TB multiply ineffective Drug therapies:
Waxy surface Lowenstein of lesion (Ghon infection) coat stimulate self- intracellularly ==> tubercle b/c intra- Isoniazid (INH).
(lipid agar (egg type) complex) destructive host formation w/ minor inflam cellular in INH requires
composed or Infect via fomites hypersensitivity (TDTH (tubercle undetect by X-ray) macrophage. catalase activity
*mycolic acid Middelbrook PPD: {cell wall} mediated) & acquired ==> hypersens response Macrophage of TB to be
fatty acid, very 7H10 serum (Purified Protein Healthy patients immunity TDTH. Tubercle is granuloma fuse to form effective. In
long chain) if agar. Simple Derivative) contain TB composed of inf macrophages, Langhans PPD+ HIV+
waxy layer medium w/ Skin test (Tine or organisms to Sulfatides (multi-acylated epithelioid cells, multincleatd giant cells. INH, liver toxic
removed can inorganic salts, Mantoux), chk in “tubercles” (mini trehalose-2-sulfates):
stain asparagines & 2-3 days. Chks lesions in lung w/ inhibit phagosome- 3.Caseating necrosis (cheesy) Granulomas 1st line drugs:
glycerol. for delayed macrophages w/ lysosome fusion similar to ==> hilar lymph n. (spreads form w/ INH,
Obligate hypersensitivity TB organism cord factor. via - lymph w/ 2 output) to epithelioid ethambutol,
Aerobe Colorless or response. intracellularly) a) lung (w/áoxygenated) cells streptomycin,
cream-colored ----------------------- Wax D (mycoside): used - Adequate CMI (TDTH) surround rifampin, para-
Looks beaded < 5 mm = (-) resp Resolves into w/ oil & water (adjuvant, response ==> get granuloma central aminosalicyclic
or granular Dry or chronic, quiescent enhance vaccine effective infect. stays contained necrosis. pyrazinamide
< 5 mm = T_s infection in elicit ab response) combinations, 6
Non-motile Wrinkled ----------------------- b) Spread via bloodstream, to Prevention!! months, Vit B6
5-9mm=try again Spreads through Lipoarabinomannin: miliary TB kill . Shot-gun Vents, UV
Cell wall: Colonies > 10mm =(+)resp out body if induce TNF alpha from pellet, lungs ,kidneys, GI. Inf. lamps, Alternative:
N-glycolym. develop in 4-8 unhealthy patient monocytes & Up to 20 yrs, inf reactivates & masks & Kanamycin,
weeks Severe TB can â b/c reactivation of macrophages develops Secondary TB respirators cycloserine, INH,
Acid/base CD4 T-Cell quiescent infection & Rifampin==
resistant Cord factor (AIDS) (secondary TB) INTRACELLULAR Primary infections: Vaccine: MDR TB is
(“serpentine pathogens, ex. chronic inf. - heal by fibrosis, calcification Intradermal, difficult to
* Catalase (+) cords” ) TB colonies Cells can airbone ----------------------------------------- w/ Live manage.
* INH (+) prod. large M. Tuberculosis survives Ghon complexes, calcified attenuated
Surface amnts NIACIN M. Bovis can under acid, basic, & dry legions visible by X-rays isolate of M. Immuno-
TB INTRA- Pellicle in only M. spread TB-like conditions but readily bovis suppressive
CELLULAR liquid culture tuberculosis!!!!! infection in killed by heat - Cavitating: inf. Blood to (Bacile of drugs reactive
humans (pasteurization). Waxy is kidney, bone marrow,brain, GI Calmette primary TB
Nitriate Red. (+) a protective layer. - Consumption: waste away Guerin
- Extrapulmonary TB: Pott’s strain or
Bactec sys: chks BCG stain)
disease, vertebral osteomyelit
release of 14CO2 (hunch back) SCROFULA =
cervical lymph nodes
Secondary
Extrapulmonary TB: Pott’s Disease (vertibal osteomyelitis)
- involvement of cervical lympth one is copying AIDS patients are s
- SCROFULA = 10% of as AIDS also bC

Non-Runyon’s mycobacteria
- M. tuberculosis
- M. bovis
- M leprae

Runyon’s classification of atypical mycobacteria


== > Pigament Formation in <==
Group Growth Rate Light Dark Species
I Slow + - M. kansasii, M. marinum
II Slow + + M. scrofulaceum
III Slow - - M. avium-intracellulare complex
IV Rapid - - M. fortuitum-chelonei complex

Cell Colony Lab Source & Virulence Factors Associated Diseases Host Treatment /
Features Features Characteristics Transmission Defenses & Prevention
Immunity
MOTTS g(+) Runyon Acid-fast stain MOTT species are 1. Mott are low virulence Group I T cell mediated Group I
Groups I-III ubiquitous in 2. produce no catalase - M. kansasii, pulmon. Inf. DTHR - Kansaii: multi-
slender rods are cultured Produce no niacin nature 3. protein antigens (killself) - M. marinum, swim pool drug + INH
long chains, aerobically 4. survives in macrophages granuloma AB ineffective - Marinum:
branching & grow very Cause disease by 5. survives in acid/basic, dry b/c bacteria are rifampin &
slowly opportunistic 6. MOTT are multi-drug Group II intracellular ethambutol
waxy infections resistant - M. scrofulaceum, cause
surface – “scrofula” Granulomas Group II
mycolic acid No person to person may form w/ - Scrofulaceum:
transmission Group III MOTT rifampin
non-motile - M. Avium Intracellulare infection
(MAC) Group III
aerobic - MAC:
Group IV ethambutol,
visualize w/ - less mycolic acid streptomycin,
acid-fast - M. smegmatis (foreskin) rifampin

Group IV
- Smegmatis,
amikacin
Cell Colony Lab Source & Virulence Factors Associated Diseases Host Treatment /
Features Features Characteristics Transmission Defenses & Prevention
Immunity
Myco- Gram (+) Grows very Acid fast staining Mammals are only Cord factor: ( Trehalose Causes pulmonary T Cell Mediated Decontaminate
bacterium slowly Of sputum reservoir b/c under dimycolate) gathers M. Bovis infection, similar to TB delayed w/ concentrated
Bovis Slender rods zoonotic. Into chains. hypersensitivity NaOH
in long Colonies Chest X-ray Infections: (TDTH)
chains- develop in 4- Infection occurs by Catalase (-) ==> INH (-), - GI tract Prevent
branching 8 weeks PPD skin test (+) ingestion of Raw resistant - scrofuloderm (skin inf) AB are development of
Milk obtained from - osteoarticular TB (joints) ineffective b/c drug-resistance,
Waxy Grown on Colonies, colorless infected animals. Proteins antigens of outer intracellular in use combo of
surface Mueller- surface stimulates self- Use of immuno- macrophage drugs.
(mycolic Hinton agar, NO NIACIN Human to human destructive host suppressive drugs
acid) Jensen- transmission can hypersentivity (steroids) re-activate Macrophage
Lowenstein Catalase (-), occur via respiratory primary M. Bovis. fuse to form
Obligate agar or resistant to droplets from Other factors enable Langhans giant
aerobe Middlebrook isoniazid, INH individuals with organism to survive in PPD + indicate exposure to cells.
7H10 serum (differs from TB) secondary TB macrophages M. Tuberculosis or M. Bovis Granulomas
Non-motile agar (INTRACELLUULAR) form w/
Detected by epitheliod
Acid-fast Cord factor Bactec within 10 Waxy layer (mycolic acid w/ surrounding
(trehalsoe days cultivation trehalose) contributes to central necrosis
Catalase (-) dimycolate) bacteria’s ability to survive
produce under acidic/basic, dry
pathogenic conditions & to resist
strains leads exposure to many
“serpentine disinfectants.
cords”&
surface Multiple Drug Resistant
pellicle in (MDR) b/c of the waxy layer
liquid or porin size.
culture.
Cell Colony Lab Source & Virulence Factors Associated Diseases Host Treatment /
Features Features Characteristics Transmission Defenses & Prevention
Immunity
Myco- Gram (+) Grows very Acid fast stain of MAC species are 1) MAC are organisms low MAC pulmonary inf. T cell mediated Multi-drug
bacterium slowly sputum ubigquitous in virulence common in AIDS patients (DTHR). Anti- therapy b/c
avium- Slender, nature (soil, dust, & 2) Do not produce catalase, body ineffective most are multi-
intracellulare rods, in long Grown on Catalase (-) throats) of normal there INH becomes resistant MAC pulmonary inf. b/c bacteria are drug resistant
chains Mueller- humans & animals 3)Protein antigens of outer similar to TB intracellular (MDR)
MAC Hinton agar Colonies are coat stimulate self-
complex Aerobic colorless & do MAC can cause destructive host hypersensit. PPD(+) may appear (-) b/c 1st line drugs:
Middlebrook not produce disease by 4) Bacterial factors enable suppressed T cell, T_s - ethambutol
Runyon Waxy surf= 7H10 serum niacin OPPORTUNISTIC organism to survive in - streptomycin
Group III mycolic acid agar (w/o infections of birds macrophages. Therefore, * Main route of inf. into - rifampin
& MOTT pigment Nitrate Red. (-) & immuno- INTRACELLULAR AIDS patients is the GI
Non-motile production) compromised 5) Bacteria survives in tract.
Chest X-ray humans, AIDS acid/basic
Acid-fast patients w/ â CD4 6)MAC are naturally mult- MAC residing in macro-
Bactec test: test for T cell. drug resistant phages INTRA-
Catalase (-) C02 production CELLULAR disseminate
from lymph nodes to
PPD skin test spleen & lungs forming
sometimes (+) lesions. May spread to all
organ systems
PPD(-) when
immuno- Tubercle (granuloma)
compromised formation in MAC inf. of
patients w/ AIDS AIDS patients is NOT
observed. Tubercle seen
in M. Tuberculosis.

Catalase (-) Catalase (+)


M. Bovis M. Tuberculosis
M. Leprae Norcadia
M. Avium Intracellulare (MAC)
Acinetobacter
Cell Colony Lab Source & Virulence Factors Associated Diseases Host Treatment /
Features Features Characteristics Transmission Defenses Prevention
Immunity
Myco- Gram (+) CAN NOT Acid-fast stain skin Humans & 1) Phenolic Glycolipid (PGL-1 Leprosy, lots of symptoms T cell Multi-drug
bacterium BE test. armadillos are the antioxidant prevent phagocyte Tuberculoid Leprosy: mediated therapy dapson
Leprae Slender, CULTURED only reservoirs (delayed type w/ rifampin
rods, long IN VITRO Lepromin test: 2) LAM: along w/ PGL-1 Borderline (intermediary) hyper-
Hansen’s chains, & similar to PPD Horizontal transfer inhibits T cell proliferation, Leprosy subdivided to sensitivity or
Disease branching But inside occurs via cause anergy (no Lepromatous Leprosy. response)
ARMADILL Catalase (-) respiratory responsiveness) The patient’s progress of the dapson w/
Waxy OS droplets disease on host’s immune In clofazimine (for
surface – & Foot pads PCR and rRNA 3)Prefers lower temp. limits tuberculoid lepromatous
mycolic of mice. probes Skin contact infection to skin, Depends on intensity of form (strong leprosy))
acid (ulcerative lesions) nasopharynx, & testicles. CMI (cellular Mediated CMI)
Lepromatous form: Leprosy is primarily a skin Immunity) response
Non-motile skin lesion biopsies Leprae likes to disease. Granulomas
& nasal secretions, grow at lower - Strong CMI: tuberculoid form w/
Aerobe look for acid fast temp. extremities 4) INTRACELLULAR - Impair CMI: lepromatous epithelioid
bacilli in ex. skin survival (no pain sensation) cells
acid-fast macrophage (foam a) Tuberculoid leprosy: surrounding
stain cells) globi strong CMI, tuberculoid central
leprosy develop w/o necrosis.
Catalase (-) Tuberculoid form: neurological involvement.
difficult to observe Disfigured (Lion’s face) Lepromatous
bacteria scars form: strong
Biopsy shows few bacilli AB response
b/c few bacteria are in but weak
lesions Activated cytokine
macrophages kill leprae. T response.
cells is detected by lepromin
test (+) Lepromatous form
not infect
b) Intermediary Leprosy
c) Lepromatous leprosy:
disfigured due to nodules
(lepromas), sense loss, pain,
& temp. Weak CMI. No
TDTH. Biopsy w/ large #
of bacilli. Respiratory
congestion immune anergy
(no activation of
macrophages) Numerous
T_s cells observed but giant
cells & epitheliods are rare.
Patient w/o hypers
lempromin(-). HIGHLY
INFECTIOUS
Cell Colony Lab Source & Virulence Factors Associated Diseases Host Treatment /
Features Features Characteristics Transmission Defenses Prevention
Immunity
Neisseria G(-) Chocolate Catalase (+) Humans ONLY Lipooligosaccharide (LOS) Male STD Gonorrhea: men N. gonorrhea Ceftriaxone
gonorrheae agar reservoir similar to LPS (+attract PMN are systemic, at the urethral. infection (expensive)
Diplococci OXIDASE (+) to primary inf site) Leads to urethritis, painful leads to local
T1-T2: pili Contains enyme Horizontal transfer urination or dysurea w/ acute Alternative:
Aerobe - T3-T5: no cytochrome C, occurs via sexual 1. Cell can add sialic acid to purulent discharge. inflammation - Spectinomycin
capnophilic pili turns black, due to contact. LOS, more C resistant due to PMN - doxycycline
as well oxidation 2. Cell wall frag are released Female STD Gonorrhea: activity &
Complement 3. Pili: attach to epithelial cells inf. vaginal or anal. complement
Non-motile ONLY glucose (+) deficiency (C5-C8) especially urogenital cells + Endocervical most common. activation.
Maltose (-) leads to meningitis, mucosal surfaces undergoes Inf. mostly asymptomatic
Lipooligo- Sucrose (-) predisposes patient ANTIGENIC VARIATION but vaginal discharge, No immunity
saccharide Lactose (-) to coccemia. 4. IgA protease: prevents IgA- dysuria + bleeding may to reinfection
(LOS) mediated opsonization on occur. Untreated results in possibly b/c
Ferments under mucous membranes salpingitis (Fallopian of antigenic
glucose, NOT (-) 5. 3 outer membrane proteins tubes), cervicitis, and pelvic variation
(omp) display antigenic inflammatory disease
Superoxol (+) test variation (PID). Inf. of liver may
(30% H202) - Protein I: Inhibit cause perihepatitis (Fritz-
phagolysomal fusion + Hugh-Curtis Syndrome) !!!
Use Thayer-Martin promote endocytosis
(TM) or (NYC) - Protein II: Neonates: purulent
agar for 48 hrs. TM Opacity protein, adhesion. conjunctivitis acquired by
==> contains Undergoes antigenic + phase newborn during passage via
vancomycin inhibt variation. Therefore makes birth canal (Ophthalmia
g(+) macrophage difficult to kill neonatorum)
Colistin inhibit: bacteria
g(-) rods - Protein III: Monoarticular arthritis
Nystatin inhibit: Porin protein, complex protein (single joint): #1 agent of
yeasts I, binds IgG blocking AB = arthritis in young adults.
Trimethroprim Rmp, blocks C mediated Found in female knees (due
inhibit: Proteus bactericidal ab f(x) to bacteremia)
6. Beta lactamase: plasmid
encoded PPNG Rash on leg!!!
7. Fe binding protiens
8. Invasiveness epithelia Disseminate Gonococcal
engulf N. gonorrhea, via Infection (DGI) ==>
INTRACELLULAR septicemia ==> LOS ==>
9. Cell wall display toxicity toxicity ==> leads to chills…
epithelial cells (tracheal skin lesions (rash) ==>
cytotoxin of B. pertussis) spreads to joints ==>
10. Autolysis/natural comp. endocarditis
11. Type IV pili: g(-) twitch,
cell specificity + phage abs
Cell Colony Lab Source & Virulence Factors Associated Diseases Host Treatment /
Features Features Characteristics Transmission Defenses & Prevention
Immunity
Neisseria G(-) Grows on Catalase (+) Humans are the 1. Capsule made of polysac: 13 Epidemic meningitis: Pili available Treat as early as
meningitides chocolate only reservoirs. serogroups A,B,C,W-135, Y youngsters & military may get possible
Diplococci agar, OXIDASE (+), provides antigenic variety, personnel. * Symptoms: activation to Penicillin G
produce only glucose & Heathly encapsulated strains resist fever, neck pain (stiffness), complement,
Aerobe transparent maltose are used individuals may phagocyt + facilitate invade headache. results Alternative:
colonies by this species be carriers of N. inflammation Chloramphenicol
Capnophilic meningitides. B = poorly immunogenic + Diagnosis: from spinal and pus
Glucose (+) resembles uropathogenic E. tap. CSF, chk á pressure, formation. Rifampin, given
Non-motile Maltose (+) 2-8% of coli’s. K1 capsule (neonatal cloudiness, culture (+),á # prophylactically
Sucrose (-) population are meningitis) E. coli ass. w/ of PMN & low glucose
Lipooligo- Lactose (-) carriers but in neonatal meningitis B capsule (confirm septic meningitis) VACCINE:
sacharide epidemic year rich sialic acid resid. polyvalent
(LOS) Needs & uses Fe will increase to Meningococcemia: upper capsular antigen
similar LPS 40-90%, Pili: attach to epithelial (non- respiratory infection, result
colonizes the ciliated) cells. Antigenic in meningitis. Sepsis is New vaccine
Naturally nasopharynx. variation diagnosed by skin rash. Menactra with
competent Disseminated capsule
Horizontal IgA protease: (similar to Intracellular Collapse conjugated to
transfer occurs gonorrheae) prevent IgA (DIC) ==> rapidly fatal. vaccine.
via respir. opsonization on mucosal
droplets & surfaces Waterhouse-Friderichsen
requires Syndrome!!!: rapid DIC,
“intimate” Beta lactamase: plasmid vascular collapse, shock &
contact (ass. w/ encoded death w/in 6-8 hr &
xchange of respir adrenal bilateral
secretions) Iron-binding protein: hemorrhage. FATAL.
scavenging Fe from transferin,
Activation of lactoferrin of host Meningococcal
infection in pneumonia: w/ carriers,
carrier due to Protein II ONLY!!! = Opacity pneumonia after viral
depressed Protein, involved in adhesion. infection.
immune Undergoes antigenic + phase
response. variation

Facultative intracellular
pathogen: enter phagocytic
vesicles & avoids death. Similar
to N. Gonorrhea in non-ciliated
mucosal cells

Type IV pili: adhesion, twitch


motility

Intracellular: into m.phage


Not on Cell Colony Lab Source & Virulence Factors Associated Diseases Host Treatment /
final!!! Features Features Characteristics Transmission Defenses & Prevention
Immunity
Eikenella G(-) Grows on Catalase (-) Human mucous 1. LPS (endotoxin) 1. Human BITE NA Augmentin
corrodens blood agar Oxidase (+) membranes 2. Pili: attach to epithelial cell WOUND!!! Could also be (clavulanic &
Slender rods that shows (mouth & upper 3. Type IV pili: adhesion to host mixed infections (staph & amoxicillin)
“pitting” of No sugar is used respiratory tract) cell surface, twitching strep) Clenched fist.
Faculatative the agar (lack oxidative & are reservoirs Associated w/ lower Alternative:
anaerobe fermentative) extremity infections by Tetracycline or
Bleach like Normal flora stepping something quinolones
Capnophilic odor on Aerobic growth contaminated to break the
agar requires hemin skin.
LPS supplementation
2. post-surgical infections:
Twitching Lysine soft tissues w/ abscesses &
motility decarboxylase (+) arthritis, empyema often
seen. Spreads throughout
Urease (-) body. Associated w/
meningitis, endocarditis,
Gelatinase (-) osteomyelitis & soft tissue
abscesses
Indole (-)

Nitrate reduction
to nitrite only

Yellow pigment
production
Cell Colony Lab Source & Virulence Factors Associated Diseases Host Treatment /
Features Features Characteristics Transmission Defenses & Prevention
Immunity
Acinetobacter G(-) Grows on Oxidase (-) Widely distributed in 1. LPS (endotoxin) Acinetobacter species are NA TMP-SMA
Baumannii blood agar nature (free living 2. drug resistance(s) hospital acquired in warm (Bactrim) similar
Coccobacilli Does not ferment saprophyte) seasons S. Saprophyticus
Produce sugars but
Obligate colonies that glucose oxidized Often associated w/ Post-surgical infections of Alternative:
aerobe are non- hospitals soft tissue with abscesses is - Kanamycin,
hemolytic often seen. - Colistin,
Non-motile Organism is able to - Tetracycline
survive on moist Organism may be also
Oxidase (-) surfaces & skin involved w/ respiratory tract
infections and urinary tract ----------------------
Non- Acinetobacter is only infections. Resistant to
hemolytic 2nd to the non- Penicillin b/c of
fermenting beta-lactamase
Glucosa Pseudomonas
oxidized aeruginosa in causing
nosocomial infections
– maybe associated w/
hospital outbreak

Ex. Acinetobacter,
Xanthomonas,
pseudomonas, can
colonize mechnical
ventilator ==>
introduce into
respiratory tract of
patient undergoing
assisted ventilation
G (-) bacilli Cell Colony Features Lab Source & Virulence Factors Associated Host Treatment /
Features Characteristic Transmission Diseases Defenses Prevention
Immunity
Bordetella G(-) Phase I – most Nasopharyngeal Humans, the 1. Capsule: A good marker for 3 stages of Ab made Erythromycin
pertussis virulent grows on swab ==> only reservoirs!!! vaccine effectiveness but ab is whooping cough or against
Coccobacilli Bordet-Gengou incubate 10 days not protective. pertussis (intense capsule. Alternative:
(WHOOPING (contains potatoe Horizontal 2. Exotoxin cough) Infection is Tetracycline
COUGH!) Singly or starch, glycerol, & Oxidase (+) transmission is 3. PERTUSSIS TOXIN (PT) is non-invasive chloramphenicol
pairs, chairs 50% blood) has a by respiratory heat-labile binary toxin. 1. Catarrhal stage: w/ bacteria
pili & O-antigen are Direct droplets among Deactivates inhibitory GTP Most infectious, remaining in Paroxymal stage:
Obligate expressed fluorescent Ab children binding* proteins (Gi) by ADP- most bacteria. respiratory oxygen therapy
aerobe test (DFA) ribosylating them Most contagious. tract. may be effective
Produce small HIGHLY Active B (adhesion factor) RRHEA & steroids â
NO carbo- “pearl-like” or ELISA CONTAGIOUS composed 5 unique polypeptides 2. Paroxymal stage: severity.
hydrate “metallic” colonies (90% gets & mediates w/ A subunit, violent cough,
fermentation infected) results in á cAMP. Toxin blocks vomit. CNS Vaccination:
& oxides Produce small zone PMN & inhibits movement by damage. Less Kills phase I
amino acids of hemolysis chemotaxis & activates contagious. bacteria, DPT
pancreatic insulin production by VIOLENT stage vaccine w/
Non-motile á CO2 help growth Islet cells. 3. Convalescent boosters 4,6, 18
Cyclolysin: dual f(x) toxin, stage: gradual months.
Requires: Cell sensitive to hemolsyin + adenylate cyclase reduction in cough,
nicotinic acid heat, dry, & chem.. 4. Adenylate Cyclase: inhibit cell may last 1-6months This vaccine has
Blood, helps f(x), especially WBC, edema been replaced by
neutralize inhibitory factor of B. anthracis. ACELLULAR
effects of fatty acid, Hemolysin: inhibit leukocyte vaccine w/
sulfides & H202 chemotaxis, phago & killing reduced side
5. tracheal cytotoxin: effects. PT,
Phase II & II less peptidoglycan frag, damages FHA, pertactin,
virulent. No blood ciliated cells. Stim IL-1 release & pili are
on phase IV. 6. Adhesion factors: pili commonly used.
(fimbriae) display phase
VIR gene variation on/off
7. Filamentous Hemagglutinin
Phase variation (FHA) attach to ciliate epithelial
(exp. on/off gene) cells
8. Pertactin: adhesion factor
9. LPS = endotoxin Lipid A & X,
X more potent!!!
** B. bronchispetica: motile, causes cough in dogs, kennel cough, unique-nitrate reduction (+), mild symptoms
Phase Description
Phase I Pili & O-antigen expressed
Phase II & III Bacteria express different antigens, less virulent
Phase IV Pili & O-antigen NOT expressed. Avirulent
G (-) bacilli Cell Colony Features Lab Source & Virulence Factors Associated Host Treatment /
Features Characteristic Transmission Diseases Defenses Prevention
Immunity
Haemophilus G(-) Grows on chocolate Catalase (+) Humans are 1. Capsule: Type b is most Nasopharyngitis: Neonate got Ceftriaxone or
influenze agar producing only reservoir commonly associated w/ H. influenzae ab from cefotaxime
Coccobacilli small colonies that Fermentation rxn for H. disease including meningitis infections (include mother &
bipolar stain are non-hemolytic are variable influenzae. (90% caused by strains) Type b non-typable or was Combination of:
contrast to (glucose only) capsule is composed of non-encapsulated protected for Cefotaxime w/
chains/filam Non- polyribitol phosphate (PRP) strains) Infection first few ampicillin
ents Beta-Hemolytic: Tryptophanase (+) encapsulated Capsules can elicit Quellung may become months of
- H. Ducreyi variants are rxn (serotyping) sinusitis, otitis life. Alternative:
facultative - H. hemolyticus Nitrates used as considered to be media or cellulites Eventually, (sensitive Pen.)
anaerobic terminal electron normal flora of 2. IgA protease: prevents also pneumonia, reproduce chloramphenicol
Capsule: Quellung acceptors the pharynx & opsonization by IgA chronic bronchitis, it’s own.
Capnophilic Rxn, destroyed by conjuctiva. epiglottitis.
CO2 autolysis by Vancomycin Carrier rate is 3. Beta lactamase: encoded on Vaccine:
endogenous enzyme sensitive ~30% plasmid, confers resistance to May lead to Composed of
Non-motile penicillin bacteremia & septic polyribitol
Requires: (hexe, navy) Requires X & V Hortizontal arthritis phosphate (PRP)
Naturally - Factor X: HEME factors and CO2 transmission 4. Pili: adhesion factor, not conjugated to
competent!!! - Factor V: NAD primarily well characterized Epiglottitis: protein. Used as
Specimens: CSF, respiratory inflamed epiglotis a booster
grows on chocolate blood & carry out droplets 5. LOS swollen, red,
agar antigenic typing of edematous tissue
capsule (Quellung Individuals had leads to airway
H. influenzae rxn) protective ab to obstruction
recovered during type b capsule by
primary isolation on age 5-6 Meningitis: H.
standard blood influenzae (type b)
agar. Also grows in Carriage is #1 cause of
close proximity to S. mostly by non- meningitis in
Aureus (beta encapsulated young children - 6
hemolytic) strains, maybe
non-typable Purulent
Satellite encapsulated conjunctivitis
phenomenon: test strains. Lead to (pink eye): caused
for nutritional req. localized by H. aegypticus
infections, w/o (Koch’s Weeks
dissemination bacillus)
G (-) bacilli Cell Colony Features Lab Source & Virulence Associated Diseases Host Treatment /
Features Characteristic Transmission Factors Defenses Prevention
Immunity
Haemophilus G(-) Grows on blood Catalase (+) Humans are the 1. LPS H. ducreyi causes SOFT chancre na Erythromycin
ducreyi agar produce small only reservoir – (endotoxin) (chancroid), painful, gential
Coccobacilli colonies that are Glucose is in genital tracts lesions & lymphadenopathy Alternative:
bipolar stain hemolytic fermented (enlarged lymph nodes). Enters Sulfa drugs
Hortizontal via break in skin. Can carry to (sulfonamide) or
facultative Requires only heme Requires Factor X transmission inguinal lymph nodes & streptomycin
anaerobic (Factor X) no need (heme) occurs via sexual multiply. May result in massive
NAD b/c can contact swelling. Highly contagious.
Non-motile synthesize Vancomycin
resistant

Exam Question: What is the causative agent of cahncroid? H. ducreyi

G (-) bacilli Cell Colony Features Lab Source & Virulence Associated Diseases Host Treatment /
Features Characteristic Transmission Factors Defenses Prevention
Immunity
Klebsiella G(-) Grows on blood Oxidase (-) Widely distributed Capsule: resist 1. Lobar pneumonia & Ab made Cefotaxime &
pneumoniae agar produce slimy in nature (soil, phagocytosis, bronchopneumonia: associated against gentamycin
Large & long – mucoid colonies Slow fermentation vegetables) attracts w/ hospital acquisition by capsule
(Friedlander’s bacilli b/c of capsule, non- of glucose, lactose macrophages alcoholics, diabetics, & patients Sensitive Pen:
bacillus) hemolytic. & sucrose (pink Often associated to area w/ chronic pulmonary disease. TMP-SMZ
facultative “Spreading” colonies on w/ nosocomial Symptoms include: fever, (Bactrim)
anaerobic MacConkey infections: Urease: productive cough, empyema,
lactose agar) develop UTI hemoptysis (spit blood) & thick
Non-motile K. pneumoniae is “currant jelly” sputum.
Bile salt selectin normal flora of R plasmid:
human colon. codes for Pen. 2. Urinary Tract Infection:
Indole (-) & common in hospital settings, b/c
Nosocomial aminoglycosid compromised by surgery,
Methyl red (-) infections: e resistance catheters, bladder retention etc…
organism is
Voges-Proskauer transmitted by LPS, endotoxin 3. Bacteremia caused by
(+) dwelling catheters Klebsiella & E. Coli in hospital
& endotracheal ST & LT settings.
Citrate (+) tubes enterotoxin
4. Tropical sprue
Urease (+) Mostly in immuno-
compromised,
H2S (-) hospital patients

Capsule: Quellng Diabetics á risk


Rxn
G (-) bacilli Cell Colony Features Lab Source & Virulence Associated Diseases Host Treatment /
Features Characteristic Transmission Factors Defenses Prevention
Immunity
Legionella G(-) Grows very slowly Catalase (+) NO horizontal LPS (endotoxin) 1. Legionnaire’s Disease: Ab are Erythomycin or
Pneumophila on BCYE (buffered transmission btw Atypical pneumonia. ineffective other macrolide
Catalase (+) charcoal yeast Superoxide humans to humans Inhibition of fusion Non-productive cough, against antibiotic
extract) agar dismutase (+) (not infectious) of phagosome & classic “shallow” sound intracellular
Thin rods produce small lysosome in when pertuss lungs in bacterium. Cephalosporins
iridescent colonies Produces brown Air conditioning macrophages & back. & amino-
Seen w/ Ag pigment in vitro tanks, whirlpool monocytes Cell glycosides
impregnatio Colonies have baths, humidifiers, INTRACELLULAR Infections may be mediated
n w/ Starry- brown pigmentation Gelatinase (+) hot water systems, survival nosocomial or community immunity Prevention:
Warthin shower heads, & acquired (hotels or cruise most Very hot water
procedure BCYE +Cysteine + Oxidase (+) contaminated Production by L. ship) important & flashing! 80C for
Fe gets L. water supply pneumophila of activation of 30 min or use
Motile pneumophila to No fermentation of systems proteolytic enzymes: 2. Pontiac Fever: fever, macrophage UV light.
grow sugars. chills, nausea, &
Aerobic Can be found in Cytotoxin: blocks headaches develop w/in Acquired Chlorination is
Bastards can hide hospitals & PMN oxidative burst 48 hrs after infection (flu- immunity not very
Grow w/ in amoeba difficult community settings like symptoms). Occurs obtained effective since L.
CO2 to kill, even w/ Capsule = F1 fraction more often in younger (protect pneumophila
Chlorine Survives patients. against survives by
intracellularly in Phosphatase: blocks subsequent colonizing
amoebae superoxide anion infection) water-dwelling
production by amebae
Transmission
stimulated PMN
occurs by
inhalation of
Hemolysin
aerosols &
(legolysin)
aspiration of
contaminated
MIP gene product
water
(macrophage
infectivity
* patient w/
potentiator) promotes
immuno-
phagocytosis by
suppressive drugs,
binding to a
smoking, or had
complement factor
surgery are at high
risk.
Beta lactamase
G (-) bacilli Cell Colony Lab Charact. Source & Virulence Associated Host Treatment /
EQ! Features Features Transmission Factors Diseases Defenses Prevention
Immun.
Pseudomonas G(-) Has Blue- Oxidase (+) Widely Exotoxin A (diphtheria toxin- 1. Necrotizing Immuno- Multi-drug
aeruginosa green color & distributed in like toxin) A subunit has ADP- bronchopneumonia: competent treatment
Long & thin grape-like Catalase (+) nature (soil & ribosyl transferase activity & B fever, cough, patients (Kirby-Bauer
bacilli in odor water) often subunit binds to cell. Leads to purulent sputum, & have test, antibiotic
chains or No sugar associated w/ inhibit protein synthesis. lung abscesses. resist. Profile)
singles Strong beta fermentation but moist areas in Target cell: heart & liver. Intact skin 1. Gentamycin
hemolysis glucose is hospitals 2. Burn wound infs: important + azlocillin
Can’t carry out oxidized Exoenzyme S: mediates burn black blue wound, to resist this 2. Augmentin
fermendation, Many will May be found in wound inf & lung inf of cystic fruity smell spreads organism.
can use any fluoresce Triple sugar iron swimming pool fibrosis (CF) into adjacent healthy Sensitive Pen:
carbon source under UV (TSI) (-) & whirlpools tissues. #1 problem No Aztreonam
oxidatively light due to which are Hemolysins: disrupt in burn patients. Use immunity
pyoverdin Citrate (+) inadequately membrane lipids & acts Wood’s UV to see P. to re- For meningitis:
Opportunist (fluorescein) chlorinated, & synergistically w/ aeruginosa. Seen in infection Ceftazidime
production Nitrate red. (+) raw vegetables phospholipase post-surgical wound
Resistant to infections. UTI & Respir.
many Fruity (grape- Nosocomial Phospholipase C: disrupts Disease
chemical like) odor infections membranes & rhamnolipid 3. UTI: Quinolone
disinfectants (inhibits ciliary action in catheterization
Pyocyanin Found to exist in respiratory Prevent burn
Associated w/ production disinfectant & 4. Sepsis/bacteremia: wound infect :
resistance to (blue-green eyewash Lipases & lecithinases - septic shock & death. Daily wound
many pigment) solutions. destroys tissue & blood cell, 5. Corneal keratitis: debridgement
antibiotics. inflam contact lens lead to use topical Ag
Produce a Beta-hemolysis blindness sulfadiazine
biofilm which 6. Otitis externa
Mucoid ExPoly. (MEP) =
contributes to (swimmer’s ear): Disinfect
alginate, form biofilms:adhere,
antibiotic cause otitis media whirlpools w/
immbolize organism. Biofilm,
resistant & 7. Pseudomonas iodine
resist. to drugs & disinfectants
ability to grow folliculitis: skin disinfectants,
(iodine). Glycocalx resist
in presence of abrasions, un- not recomm.
phagocytosis. Collagenase:
disinfectants chlorinated hot tubes b/c organism
degrade collagen Elastase;
cleaves IgA, IgG, & survives inside
complement. Alkaline 8. Osteochondritis: biofilm.
protease. puncture wound of
PYOYANIN: blue-green foot. Tennis shoes Chlorination
pigment w/ bacteriocidal effective to kill
activity. Pyoverdine = 9. Podiatrically pseudomonas
fluorescein (green-yellow UV) important – infect P. Aeruginos
Multiple Antibiotic Resistance: nail. Eats nail & turn sensitive to â
beta-lactamase, acetylating into green goo pH. Acetic acid
enzymes. (vinegar)!!!
#1 (most common) non-fermenting g(-), bacillus nosocomial infectious disease agent
Cell Colony Features Lab Source & Virulence Associated Diseases Host Treatment /
Features Characteristic Transmission Factors Defenses Prevention
Immunity
Mycolplasma Smallest, Growth leads to Use gram staining Humans are only 1. Antimicrobial 1. Atypical pneuomia Ab & T cells Doxycycline
pneumoniae free-living “mulberry” colonies to rule out any reservoir resistance: b/c of lack (walking pneuomonia) important (leads to yellow
organisms w/ older colonies other causative of cell wall, innately most common cause, teeth in children)
(EATON’s “fried egg” colony, agents Horizontal penicillin resistant. infection is mild or Associated
agent & Pliable slow growth transmission via Does not make cell aymptomatic but can be w/ auto-ab Erythromycin
PPLO) Glucose respiratory wall fatal. Chest x-ray show production for children &
Cell fermentation(+) droplets mostly unilateral lower lobar (IgM cold preg. women
membrane, among teens & 2. Adhesion Protein involvement agglutinin)
contain Tetrazolium dye young adults. P1: mediates Symptoms: interstitial
sterol reduction: turn adhesion to epithelial pneumonia w/ non- Cytokines
blue to yellow cells, ciliated cells & productive (persistant) activated
No cell wall RBC cough & inspiratory EXCEPT
Detected: crackles, fever, chills, IL-2
G(null) Stain w/ 3. Production of H202 headache & chest pain
fluorescent labeled possibly contributes Complictions: lead to CNS
Aerobic ab, ELISA; latex to mucosal damage & & heart complications
agglutination damage to RBC
Motile 2. Raynaud’s
4. Mycoplasma, phenomenon occur to
Special survive intracellulary, cold-agglutination
nutrient req. cause most damage antibodies, leads to
cholesterol extracellularly necrosis of fingers & toes
cell if in sickle cell anemia
membrane patients

3. Endocarditis (inflame.
Of heart muscles)

4. Guillain-Barre Syndr.
(asc paralysis more
commonly associated w/
Campylobacter infection)
G (-) bacilli Cell Colony Features Lab Source & Virulence Associated Host Treatment /
Features Characteristic Transmission Factors Diseases Defenses Prevention
Immunity
Helicobacter G(-) Organism grows on Oxidase (+) Human GI tract is 1. LPS 1. Chronic Immuno- Combination of
pylori selective agar Catalase (+) reservoir. Oral 2. Motility – burrow through Gastritis: competent drugs:
Pleomorphic TSI (-) fecal route is mucin layer stomach lining Causative agent!!! patients have
(s-shaped, Chocolate agar or Urease (+) involved in Acute infection of chronic Pepto-bismol
bacilli, modified Thayer- Nalidixic acid horiztonal 3. Resistance to stomach stomach inflammator (Bismuth salts),
curved rods, Martin incubate for resist. transmission acidity enhanced by epithelium. y response amoxicillin, &
spirochete) a > 1 week under Nitrate reduct. (-) (human to human) movement of Helicobacter Organism causes w/ metronidazole.
microaerophilic Hippurate into & within the protective superficial mucosal moncytes,
Produce: conditions hydrolysis (-) mucous layer of the stomach inflammation & is macrophages Pen. Sensitive:
corkscrew (chocolatization Cephalothin sensit. (not epithelia). non-invasive. & Sub. Amox w/
motility detoxifies agar) Microaerophilic nature Symptoms: nausea, lymphocytes Tetracycline
Endoscopy: detect organism helps survival abdominal in stomach
Micro- by biopsy. discomfort. mucosa. Also include
aerophilic 4. Enzymes: proton pump
(grow in 6% 14C-urea in food & a) Mucinase causes 2. Duodenal peptic Antibody inhibitor ex.
O2, 10% look for 14CO2 in breakthrough of mucous ulcer: (#1 causative formation omeprazole
CO2) patient’s breath & layer in stomach (resist acid) agent) follows not
somewhat serology b) Urease: stomach lining chronic gastritis. protective
tolerant to produce small amounts of Symptoms: therefore
stomach urea .. leads to ulceration burning abdominal patients after
acidity pain 1-3hr after treatment
5. Adhesion factors: attach meals that may be may relapse
to stomach (resist peristalsis) relieved by eating
& antacids.
6. Hemolysin (128kD) found Complications
w/ cytotoxin protein include: bleeding,
& perforation of GI
7. Vacuolating toxin: 50% tract
isolates á virulence of strain
(pathogenicity island* á 3. Gastric
virulence, carcinoma: chronic
gastritis
8.Cag protein: stimulates
host prod. of IL-8 & other
signal transduction events

9. Catalase / superoxide
dismutase: protects from
intracellular killing phago.
10. Produce acid inhibitory
protein (induce
hypochlorhydrin)
G (-) bacilli Cell Colony Lab Source & Virulence Associated Host Treatment /
Features Features Characteristic Transmission Factors Diseases Defenses Prevention
Immunity
Vibrio g(-) Grows blood Oxidase (+) Human GI tract is Cholera toxin (CT) A subunit Explosive Diarrhea! Large Immediate &
cholerae agar Lactose (+) reservoir for this (ADP-ribosylates) cytoplasmic G Cholera, remains in inoculum continuous IV
comma Sucrose (+) organism protein (G_s) regulates host SI (duodenum) & needed to or fluid
shaped Enrich in Indole (+) adenylate cyclase. á cAMP non-invasive. infect replacement
bacilli highly Methyl Red (+) Transmitted by oral- levels, induce intestinal cells to along w/
(curved alkaline Urease (-) fecal route (by release Cl- ions into lumen. “rice water” diarrhea electroylates
rods) medium drinking water & Toxin inhibits Na+ absorp, á (large vol. – 1
(loves higher Chk rice-water intaking raw & ions, water release (diarrhea!) liter/hr) “rice”: is Replenish w/
motile pH) stool for motile undercooked seafood, mucus from NaCl / KCl –
(flagella) bacteria, culture on unpeeled fruits) LPS: endotoxin, 6 serotypes intestinal wall. add glucose to
Thiosulfate- TCBS media & see improve
Citrate-Bile- if yellow b/c V. Hortizontal 01: NON-INVASIVE cause No fever involved uptake of salts
Sucrose cholerae ferment transmission amongst epidemics, no age preference but by intestinal
(TCBS) agar sucrose humans usually in children, preg. Untreated patients cells. (try
Women, long-term immunity. may die in hrs due to Gatorade!)
Can grow Large inoculum hypotension,
w/o salt (10^7 – 10^11 cells) Non-01: sporadic infections dehydration, & Tetracycline is
b/c are killed by shock. used for severe
HALOPHIC stomach acidity 0139: INVASIVE infection in cases
indiv. > 40 yrs, no immunity Complications:
Exist in salt water for gained, possible reinfection. - electrolyte
long periods of time / imbalance
found associated w/ Flagelluem (H antigen): an - metabolic acidosis
plankton, shellfish, adherence factor. Motility - hypoglycemia: kid
bacteria to enter into mucous - abortions
layer, non-motile is avirulent.
Dehydrationleads to:
Adhesion factors: mediate - sunken eyes
bacterial adherence to SI - loss of skin turgor
epithelium. AF + CT to cause - patient, comatose
full-blown disease - death

TCP-ACF: Toxin Co-regulate


Pilis pathogenicity island.

Acidic polysacc. Capsule found


ONLY in 0139 strain. It is
INVASIVE. Can go out of
intestine & cause infection
Test question: Given tainted water w/ cholera w/ a dosage of 100 organisms, would a normal person be sick? No b/c it requires a lot more, but Shigella could kill you
Cell Colony Lab Source & Virulence Factors Associated Host Treatment /
Features Features Characteristic Transmission Diseases Defenses Prevention
Immunity
Enter- g(-) rod in Grows on Oxidase (-) Most E.coli are 1. Heat-labile enterotoxin (LT): Enteritis or sIgA against Fluid &
otoxigenic pairs & almost any Lactose (+) harmless cholera-like binary toxin Traveller’s diarrhea CFA & other electrolyte
chains medium, Sucrose (+) commensals of GI encoded in ENT plamids. It is (Turista). Non- bacterial replacement :
Escherichia forms large ------------------------- tract cytotonic! Active A subunit invasive & remains in components NaCl, KCL,
Coli (E.Coli) anaerobic gray colonies, Indole (+) ADP-ribosylates cytoplasmic G the lumen of SI. combats glucose, &
on blood Methyl red (+) Some E.coli contain protein (G_s) regulates host infection. water.
AKA (ETEC) strains are agar, some VogesProskauer (-) extrachromosomal adenylate cyclase. á cAMP. Symptoms: mild,
motile strains are Citrate (-) DNA such as large (diarrhea!) watery diarrhea Antibiotics not
“Dr. hemolytic {IMViC assays} ENT plasmids which similar to V. Cholera given
Trachman’s ==> unique E.Coli enable E.coli to be 2. Heat-stable toxin (ST): EXCEPT the mucus. prophylaxively
favorite pet” ------------------------- pathogenic. encoded by ENT plasmids, but use
TSI – butt & slant activates guanylate cyclase & á norfloxacin .
yellow w/ gas Enterotoxigenic E.coli cGMP levels. Blocks ion uptake Pepto-bismol
prod. can colonize small from lumen. (bismuth salts
intestines. are inhibitory)
H2S (-) Heat Looks like Rxn
Oral-fecal route by LT Cholera á cAMP
Serology: consuming ST Yersinia E. á cGMP
Kaufman – White contaminated water --------------------------------------------
Scheme LPS:endotoxin, Oag, serum resist
REMEMBER: KOH Hortizontal
K = capsule, fibriae transmission from Flagellum: H,ag bacteria enter
O = LPS somatic human to human
mucus layer. w/ phase variation
H = H ag, flagella
TRAVELLER’S
Pili: attachment to host cells.
Culture on DIARRHEA or AKA
2 Types Mannose
MacConkey- TURISTA
Common pili Sensitive
lactose medium &
P pili Resistant
blood agar
Sex pili: conjugative plasmids
transfer F+ plasmid or Hfr
(donor, male) to F- (female)
Fimbriae: attachment to host
cells SI (tissue specific) CFA
(colonization factor antigen)
Antibiotic & heavy metal resist:
Encode on conjugative plasmids
Capsule: protect from acidicity
E. coli produce hemolysins
E. coli produce siderophores (Fe
acquisition)
Cell Colony Lab Source & Virulence Factors Associated Host Treatment /
Features Features Characteristic Transmission Diseases Defenses Prevention
Immunity
Entero- g(-) Grows on Oxidase (-) Human GI tract is Enterotoxins: lysogenic phage Hemorrhagic colitis: Secretory Fluid &
hemorrhagic almost any Lactose (+) reservoir. Oral-fecal - Shiga-like toxin I (SLT-1): stays in lumen of LI & IgA against electrolyte
E. Coli bacilli pairs agar forming Indole (+) route by drinking toxin f(x) in LI. Results: cleavage non-invasive. Blood bacterial replacement.
or chains large gray Methyl (+) contaminated water of euk 28s ribosomal RNA (60S). diarrhea. Blood due components
AKA (EHEC) colonies Urease (-) & undercooked Mucosal cells poisoned, inhibits to cytokine activity combats Antibiotics not
motile TSI (-) for H2S protein synthesis. Bloody stools. combined w/ SLT. infection. used for
Citrate (-) Consume Enterotoxins enter bloodstream Non- prophylaxis.
anaerobic VogesProskauer (-) unpasteurized fruit may be associated w/ kidney Hemolytic-Uremic inflammator
SORBITAL util (-) juices, playing in damage. Syndrome: result in y response Prevention:
Serology 0157H7 contaminated water acute renal failure, to infection. - well cooked
- SLT-II (STX) : both of these are hemolytic anemia & Non- meals, burgers
called VEROTOXIN b/c of their thrombocytopenia, invasive & pasteurized
toxic effects on vero monkey 5% of cases. w/ â fruit juices
kidney cells. platelet numbers.
Complication Better treat:
- Enterohemolysin (sequelae) seen in - fluids + salts
- LPS (endotoxin) children & in - chlorinate
- Pili: Adhesion, mediate immuno- water
bacterial (attachment to large compromised or
intestinal epithelium. Plasmid elderly.
encoded
- capsule (possibly)

- ACID RESISTANT, ~50


bacteria to cause disease (like
Shigella). EHEC more acid resist

- EAE genes (similar to EPEC) >


attaching & effacing lesions

Non-invasive Invasive
Lactose Ferm. sugar Organisms
- ETEC - EHEC Occurs E.Coli, Klebsiella, Enterobacter
- Vibrio cholerae - Shigella Not occur Shigella, Salmonella, Proteus, Pseudomonas (Acinetobacter)
- EPEC - Salmonella Occurs slowly Serratia, Vibrio

I M V C
E.Coli + + - -
Enterobacter - - + +
aerogenes
Cell Colony Lab Source & Virulence Factors Associated Host Treatment /
Features Features Characteristic Transmission Diseases Defenses Prevention
Immunity
Salmonella g(-) Grows on Oxidase (-) Animal & human GI LPS = exotoxin: mediates Non-typical Strong Most often is
Typhimurium almost any Lactose (-) tracts are reservoirs endotoxic shock, involves Salmonellosis inflammatio self-limiting
rod in pairs agar medium Indole (-) (poultry, reptiles such inflammatory response of (Enterco-colitis): n response
AKA or chains large Methyl red (+) as turtles & iguana). intestinal mucosa. S. Resistant Initial invasion of w/ PMN & Treatment
VogesProskauer (-) Oral-fecal route by mucosal cells of T-Cells. enterocolitis:
(S. enteritidis anaerobic Indole (-) consume Invasion factor(s): bacteria ileum & large CMI is - fluid &
or S. enterica) Citrate (+) contaminated water invades host’s mucosal cells of intestines may be important electrolyte
motile TSI (+) H2S or food involved in the ileum & LI. Organism is able followed by systemic b/c of replacement
Urease (-) hort. Transmission. to survive in macrophages, not invasion. intracellular
PMN. location. Antibiotics
Serology: Use antacids Symptoms: FEVER, don’t eliminate
Kaufman – White predispose for Flagellum (H ag): strong nausea, water Salmonella
Scheme infection. motility enable bacteria to enter diarrhea, headache, from GI but â
REMEMBER: KOH into mucus layer. enteric inflammation, duration.
K = capsule, fibriae Chronic carrier status & pus in stools.
O = LPS somatic Inhibition of phagosome- Ciproflaxin,
H = H ag, flagella Cutting lysosome fusion: salmonella Entercocolitis may TMP-SMZ
boards/knives (need multiply in macrophage, then mimic appendicitis (bactrim) used
Culture stools on to clean w/ bleach) lyse macrophage & spread to for systemic
selective medium nearby cells. Bacteremia: intestines infections
Raw eggs to blood stream.
Capsule (K ag): help bacteria Leads to endocarditis, Avoid raw
Survives freezing survive stomach acidity bile duct infection, eggs & use of
water, food or water septic arthritis, antacids
for several weeks Multi-drug resistant factors pneumonia. AIDS
patient may have Wash hands
Enterotoxin: epidermal growth recurrent bacteremia before eating
factor (EGFR) on host cell to
bind to, leads to á Ca in cell. á Osteomyelitis:
leukotriene synthesis opens up salmonella bone
Ca channels lead to rearrange infections arising
actin. Results: leukotrienes á from bacteremia seen
inflammation. in sickle cell anemia
patients
VIR genes turned on inside host
cell ==> antigenic variation Post-inf. Reiters
Syndrom: arthritis,
Pathogenicity island (SP-1, SP-2) HLA-B27

Expression of genes can adapt to


acidic environment
Which organisms are associated w/ Reiter’s syndrome? Salmonella typhimurium & typhi
Cell Colony Lab Source & Virulence Factors Associated Host Treatment
Features Features Characteristic Transmission Diseases Def.Im. / Prevent
Salmonella g(-) Grows on Oxidase (-) Human GI tract is Vi antigen (capsule, hides Typhoid Salmonellosis Stomach Antibiotics
Typhi almost any Lactose (-) main reservoir for surface markers, removed by (Enteric fever) & acid kills ciproflaxin
rod in pairs agar medium Indole (-) these organisms boiling water). Bacteria resists paratyphoid fever. 2 Salmonella.
AKA or chains forms large Methyl red (+) stages: Large Systemic Inf:
gray colonies VogesProskauer (-) Oral-fecal route by LPS = endotoxin: major VF in amounts of - TMP-SMZ
S. paratyhpi anaerobic Citrate (+) consuming mediating endotoxic shock, Invades Peyer’s patch of inoculum (Bactrim)
TSI (-) H2S contaminated water involves in inflammatory ileum & LI infects causes the - Amoxicillin
motile Urease (-) & food. response of intestinal mucosa. various organs & infection - chloro-
May mediate serum resistance. tissues: liver, kidneys, ramphenicol
Serology: Hortizontal spleen, bone marrows,
Kaufman – White transmission Invasion factor(s): bacteria gall bladder, skin Misc notes:
Scheme invades mucosal & epithelial (ROSE SPOT TRUNK) Ciproflaxin
REMEMBER: KOH Chronic carrier status cells of ileum & LI using this & heart eliminates
K = capsule, fibriae in humans is factor. Able to survive in carrier state
O = LPS somatic possible – survive in macrophages but not PMN. Wk Issue
H = H ag, flagella GI tract w/
1-3 Incubation
predilection (bias) Inhibition of phagosome-
for gall bladder. 2-3 Hepatosleno-
DNA probe for Vi lysosome fusion: Salmonella
megaly
antigen (capsule) multiply in macrophage, then
2-3 Bradycardia
lyse macrophage & spread to
WIDAL test: look nearby cells 2-3 ROSE SPOT
on trunk
for agglutinating ab
4 Secondary
acute & Flagellum (H ag): bacteria enters
infections
convalescent stages mucus layer. Has phase var.
GI tract
(refer to B. Pertusis) similar to E.Coli
see 4x á in ab
titers, If (-) rxn then Multi-drug resistant factors: Symptoms:
inconclusive. plasmid encoded beta lactamase - FEVER
- abdominal pain
& chloramphenicol transferase.
- headache
- water diarrhea
Ability to adapt to acidic
- enteric inflammation
environment expressed by genes
- pus in stool

Bacteremia: intest. to
blood stream, subacute
endocarditis, bile duct
inf.

Reiters Syndrome:
arthritis (see S. enterica)
Abortion: enteric fever
* Most invasive! Enterics are resistant to bile
Cell Colony Lab Source & Virulence Factors Associated Host Treatment
Features Features Characteristic Transmission Diseases Defenses Prevent
Immunity
Shigella g(-) Grows on Oxidase (-) Human GI tract & GI Enterotoxin: Shiga toxin has 1. Bacillary Strong acute Infection is
species almost any Lactose (-) tracts of higher same activity. RNase activity Dysentery inflammatory self-limited.
rods in agar forming Indole (-) primates cleaves euks 28S ribosomal RNA. (shigellosis): invades response to
pairs or large gray Methyl red (+) Stops protein synthesis, results in mucosa of LI but not infection by Fluid &
chains colonies. On Urease (-) Can survive for bloody diarrhea. Verotoxin: underlying muscle PMN. electrolyte
EMB, TSI (-) for H2S months, but it is ONLY S. dysenteriae (see chart replacement.
non-motile organism is Citrate (-) delicate. below) Symptoms include: Secretory IgA
colorless or VogesProskauer (-) Watery diarrhea against Antibiotics
Lactose (-) white Five F’s: LPS (endotoxin): epithelial progresses to bloody bacterial shortens
NON-MOTILE!!! - food necrosis end of ileum & colon. diarrhea, fever, & infection duration but
NON- - fingers inflammation w/ pus not lessen
MOTILE SERENY TEST: - feces Polysach. somatic O antigen to in stools. Toxin kill CMI important intensity
(inoculate guinea - flies avoid phagocytosis, serum mucosal cells of b/c of
anaerobe pig or rabbit eyes) - fomites resistant intestine. Often intracellular Bactrim
found in children & location (TMP-SMZ)
Get specimen from Common among Invasion factors: similar to EIEC. elderly. used for
rectal swab, homosexual men Plasmid encoded & mediates serious
sigmoidoscopy invasion & destruction of 2. Hemolytic-Uremic cases, stop
epithelium of colon. Syndrome: Shigella transmission
infection may result .
Commandeering of Actin in actue renal failure, Nalidixic
filaments in host cell for hemolytic anemia, & acid or
mobility in host cell ==> helps thrombocytopenia. newer
-------------------- spread to nearby cells quinolones
Innately 3. Post-Shigellosis
non-motile but Shigella are able to survive Reiters Syndrome: Prevent
once inside the acidity of stomach: infection arthritis associated from
host becomes requires very small inoculum w/ HLA B27 consuming
motile b/c of 50-300 cells uncontaimin
actin filaments ted food &
Multi-drug resistance plasmids water

Shigella Type Serogroup # serotypes Comments


S. boydii C 18
S. flexneri B 14 Most common in developing countries
S. sonnei D 1 Most common serogroup in US, acquired when traveling to endemic area (NON-MOTILE!!!)
S. dyenteriae A 12 Most common in remainder of world, most pathogenic, most invasive, produces Shiga toxin (LACTOSE (+)!!!)

Test question: Which pair organisms are associated wit h hemolytic-uremic disease? Shigella dysenteriae & EHEC
Cell Colony Lab Source & Virulence Factors Associated Diseases Host Treatment /
Features Features Characteristic Transmission Defenses Prevention
Immunity
Bacteroides g(-) Grows Anaerobic growth, Human GI tract is 1. Polysaccharide capsule 1. Intraabdominal Both Debridgement
fragilis anaerobically penicillin resistant resvoir for (K antigen): major VF w/ Disease (abscesses): humoral & & surgical
slender on complex Bacteroides anti-phagocytic activities. opportunistic, associated CMI drainage
rods, pleo- agar Superoxide Attach to peritoneal w/ post-op. peritonitis, involved in before
morphic medium, dismutase (+) MOST numerous mesothelium inf. abdomen w/ combating antibiotic
brain heart bacterial species in intestinal contents Bacteroides treatment
obligate infusion – Catalase (+) human body 2. catalase & superoxide gunshots, stab wounds, infections - clindamycin
anaerobe forms gray dismutase: allows organism surgery, cancer, (80% by
glistening Indole (+/-) Outnumbers E. coli to remain viable for days w/ b. fragilis) UTI in women Seriously ill:
non-motile colonies due in colon. O2 (aerotolerant) lead to pelvic abscesss - chlora-
to capsular Growth in 20% bile (PID), & brain & lung mphenicol
capsule polysacch. Alcoholics, immuno- 3. Enzymes contribe to infection
Antibiotic Resistant compromised Bacteroides (act as Alternatives:
NO - colistin patients, patient w/ spreading factor): 2. Soft tissue & cellulites: - erythromycin
endotoxin - kanamycin anesthesia can lead to - heparinase for mixed infections w/ or moxalactam
activity b/c - vancoymcin pulmonary infections intravascular clotting peptostreptococcus,
it lacks (opportuntistic) - collagenase bacteroides,
Lipid A !!! - hyaluronidase fusobacterium, &
Mixed w/ - lipases actinomyces (all
an/aerobes - nucleases (DNases, anaerobes) Post-opt.,
May have synergistic RNases) cutaneous & or
interaction - Pili / fimbriae mucotaneous infections.
UTI

3. Bacteremia: may cause


endocarditis & brain
abscesses

4. Upper/Lower infection

5. Crepitant cellulites
(especially in foot),
-------------------- usually post-op inf.
Obligate
anaerobe :: 6. DIABETIC FOOT
facultaitive & INFECTIONS
anaeroebe (FECAL FALL OUT)!!!
(10,000: 1 ratio)
Cell Colony Lab Source & Virulence Factors Associated Host Treatment /
Features Features Characteristic Transmission Diseases Defenses Prevention
Immunity
Proteus g(-) Grows on Oxidase (-) Normal flora human 1. Urease: splits urea to NH4 1. Urinary tract IgA, IgG, Ampicillin for
mirabilis blood agar Lactose (-) LI (colon), crap found ions ==> alkaline urine promotes infections: a) made cystitis
pairs or forms large Indole (-) in soil & water. urolithiasis (kidney stones). pyelonephritis against
chains rods “swarming” Methyl red (+) Destroys the urinary epithelium b)cystitis. Infections various Bactrim (TMP-
colonies” w/ Urease (+) Nosocomial or & cause infection & impair may be acute or bacterial SMZ) for
facultative blood agar TSI & SIM (+) for iatrogenic infection kidney fxn chronic. components. pyelonephritis
anaerobe only w/ H2S
putrid odor Citrate (+) UTI can happen in 2. Strong motility: movies up Symptoms: PMN
highly- VogesProskauer (-) healthy & immuno- urinary tract - pain involved in
motile b/c compromised w/ - fever acute
peritrichou Phenylalanine large inoculum ==> 3. LPS (endotoxin) - pyuria infections
s flagella deaminase (+) opportunistic - hematuria (pus is
distinguish from - á urination formed)
P. vulgaris!!! Hortizontal (synchuria)
transmission Macrophage
2. Urolithiasis: Mg- s & T cells
NH4 phosphate salts respond to
form kidney stone chronic
due to NH4 infections

3. associated w/
bacteremia,
pneumonia,
endocarditis

UTI: contamination with fecal matter


a) lower UTI = cystitis, infection of urinary cyst, bladder
b) upper UTI = pyelonephritis, serior infection see blood, pus in urine
c) renal or kidney “stones” = urolithiasis (struvite) & apatite, formation due to production of urease (ie Proteus)
Cell Colony Lab Source & Virulence Factors Associated Host Treatment /
Features Features Characteristic Transmission Diseases Defenses Prevention
Immunity
Proteus g(-) Grows on Oxidase (-) Normal flora of 1. Urease: splits urea to NH4 1. Urinary tract IgA, IgG, Ampicillin for
vulgaris rod, pairs, blood agar Lactose (-) human LI (colon) ions ==> alkaline urine promotes infections: a) made cystitis
chains forms large Indole (+) urolithiasis (kidney stones). pyelonephritis against
“swarming” Methyl red (+) Nosocomial or Destroys the urinary epithelium b)cystitis. Infections various Bactrim (TMP-
facultative colonies” w/ Urease (+) iatrogenic infection & cause infection & impair may be acute or bacterial SMZ) for
anaerobe blood agar TSI & SIM (+) for kidney fxn chronic. components. pyelonephritis
only w/ H2S UTI occur in
motile b/c putrid odor Citrate (-) immuno- 2. Strong motility: movies up Symptoms: PMN
peritrichous VogesProskauer (-) compromised urinary tract - pain involved in
flagella - fever acute
Phenylalanine Hortizontal 3. LPS (endotoxin) - pyuria infections
deaminase (-) transmission - hematuria (pus is
distinguish from 4. Multiple drug resistance - á urination formed)
P. mirabilis!!! (synchuria)
Macrophage
2. Urolithiasis: Mg- s & T cells
NH4 phosphate salts respond to
form kidney stone chronic
due to NH4 infections

Proteus mirabilis Proteus Vulgaris


Indole (-) Indole (+)
Citrate (+) Citrate (-)
Phenylalanine Phenylalanine
deaminase (+) deaminase (-)
distinguish from
P. vulgaris!!!
Cell Colony Lab Source & Virulence Factors Associated Host Treatment /
Features Features Characteristic Transmission Diseases Defenses Prevention
Immunity
Europathogenic g(-) Grows on Oxidase (-) Normal flora of 1. Polysaccharide capsule (more 1. Urinary tract IgA, IgG, Ampicillin for
E. Coli (UPEC) rod, pairs, almost any Lactose (+) human LI (colon) infectious): strains w/ K1 infections: (#1 cause made cystitis
chains agar surface Sucrose (+) capsule contains K1 sialic acid of UTI, 80-90%) against
Extraintestinal E. forming Indole (+) Nosocomial or associated w/ bacterial a)pyelonephritis – various Bactrim (TMP-
coli infection facultative large gray Methyl red (+) iatrogenic infection meningitis. E. coli w/ K1 are infection of kidneys. bacterial SMZ) for
anaerobe colonies. On Urease (-) NOT immunogenic, protects Infections may be components. pyelonephritis
EMB, have TSI (-) for H2S UTI occur in against phagocytosis & against acute or chronic.
motile metallic Citrate (-) immuno- complement-mediated lysis. Most nosocomially PMN Ampicillin &
sheen (b/c compromised acquired infections involved in gentamycin -
eosin) VogesProskauer primarily 2. Hemolysins: release cytokins, are complicated & acute
(+) This unique!!! leads to inflammation present as infections
Enterobacter Hortizontal & Alpha hemo: disrupts pelonephritist. They (pus is
yields larger Vertical transmission lymphocytes are opporunitic & formed)
colonies Beta hemo: membrane-bound & infect
h& inhibits PMNs Macrophage
2. Urolithiasis: Mg- s & T cells
3. Motility: flagella (H antigen) NH4 phosphate salts respond to
b/c of phase variation form kidney stone chronic
due to NH4 immuno- infections
4. LPS (endotoxi) most inf. compromised
isolates 04.05.075
b) Bacteremia &
5. Pili sepsis: (#1 causative
a) P (PAP) pili involved in UTI, nosocomial agent)
attaches to uroepithelia. These UPEC bacteremia is
strains can cause pyelonephritis complicated.
P Pili are “mannose resistant”
c) Neonatal
Type 1 pili used for adhension meningitis (#2
(see bolow) causative nosocomial
agent) cause sepsis &
6. S. Fimbria: not well reconigzed death. Need lumbar
puncture: CSF á
7. Siderophores: chelate host Fe PMN, â glucose
released by RBC (hemolysins), &
take from transferrin & allow Fe d) Opportunistic
uptake by E. Coli pheumonia similar
ot Klebsiella
8. Serum resitant: resist pneumoniae
oposonization / destruction by
host complement factors or due
to k1 capsules
L Cell Colony Lab Source & Virulence Factors Associated Diseases Host Treatment /
theZoonotic Features Features Characteristics Transmission Defenses & Prevention
(GI infect)
Immunity
Campylo- g(-) CAMPY Oxidase (+) Animal GI tract 1. CT – like enterotoxin 1. ENTERCOLITIS: acute Acute Infection is self-
bacter agar Catalase (+) (zoonotic infection) 2. cytotoxin (not Shiga- infection of epithelium of SI inflammatory limiting but
PLEO- supplement Hippurate like) results in bloody (jejunum & ileum) & LI. response by requires fluid &
jejuni MORPHIC w/ blood & Hydrolysis (+) Contact w/ infected: diarrhea Common in children PMN. Both T- electrolyte
other animals, dogs, cats, 3. LPS (endotoxin) cell mediated replacement.
Curved nutrients Urease (-), fowl (chicken), but 4. Flagellum: bacterial 2. Enterotoxin – invades the immunity &
rods, spiral [differentiate from NOT eggs! motility; darting epithelial cell, superificially secretory IgA & Erythromycin
forms w/ Grows from helicobacter] movement IgG.
coccobacilli 4 - 42C to Even animal scratches 5. adherence factor Symptoms:
inhibit other Fecal specimen: (did you see her nails?) 6. grows well w/ bile - fever
Micro- bacteria CAMPY AGAR 7. can be invasive - nausea
aerophilic +vancomycin Improper handling: - abdominal discomfort
(6% O2 & +cefoperazone - utensils & cooking - watery diarrhea
10% CO2)
Grows @ 42C, Raw unpasterurized Severe cases:
âO2 & áCO2 milk or water - cytotoxin production
Motile (can contaminated (feces) (bloody diarrhea + pus) or
penetrate SI b/c bacteria survives in enteric inflammation & last
& multiply) milk for up to 5 weeks for about a week (self limit)
when stored at 4C - Extraintestinal disease:
“Darting” survives intracellularly 7days
motility Sensitive to chemical
disinfectants, acidity, & 2. Guillain-Barre Syndrome:
chlorination - ascending paralytic disease
- viral inf. (influenza, Epstein-
Relate to spontaneous Barr Virus EBV)
abortion(effect) infant
b/c infected 3. Abortion: organisms
genitourinary tract x- invasiveness can invade fetus
over placenta or across placenta during a
septicemia or ascending inf.
Oral-fecal route (man
to man transmission) * 4. Reiter syndrome:
in children - post-infection, arthritis,
associated w/ HLA-B27
Zoonotic Cell Colony Lab Source & Virulence Factors Associated Diseases Host Treatment /
(GI infect) Features Features Characteristics Transmission Defenses & Prevention
Immunity
Yersinia g(-) Grows on Oxidase (-) Animal GI tract is 1. V and W antigens: 1. (Hemorrhagic) Acute Infection is self-
Entero- blood agar Catalase (+) reservoir therefore unknown f(x) but Enterocolitis: Acute infection inflammatory limiting but
colitica short rod Lactose (-) zoonotic infection essential (found in Y. of the epithelium of the SI response by requires fluid &
No Urease (+) pestis) (ileum) & LI. Symptoms of GI PMN. Both T- electrolyte
facultative siderophore, Methyl red (+) Water contaminated tract include: fever, nausea, cell mediated replacement.
anaerobe so needs VogeProskauer (-) w/ animal feces may 2. Enterotoxin: YST is abdominal discomfort, immunity &
iron for also be involved. heat-stable enterotoxin headache, water diarrhea secretory IgA &
motile growth similar to ETEC (E.coli) which may be bloody + pus IgG. W/ Y.
Oral-fecal route (enteric inflammation). enterocolitica
non-motile Tolerates at involved man-man 3. LPS (endotoxin) sepsis,
at 37C or in cold storage transmission Y. enterocolitica is destroyed gentamycin of
host 4C 4. Flagellum: mediates by stomach acidity, large Bactrim used
Raw milk (also from motility outside of host, inocula 10^9 cells to be
cold-stored blood) but non-motile at 37C infected. Use good
hygiene
Y. enterocolitica is 5. Serum resistance: 2. Terminal ileitis: lead to
NOT need insect (plasmid encoded) peritonitis
vector for transmission
6. YadA gene: outer 3. Septicemia: rare; lesions
membrane protein, internal organs
adhesion to epithelial
cells 4. Reiter syndrome:
- post-infection, arthritis,
7. Genes, invasiveness: associated w/ HLA-B27
- Inv gene (invasion),
surface protein Bacteremia w/ this organism ;
- Ail gene (adherence therefore, can be blood-borne
invasion locus), by transfusions
facilitate entry into
specific cells

8. FACULTATIVE
INTRACELLULAR

9. Pathogenicity island

10. Type III – secrete


proteins

VF regulated by
calcium & by
temperature
Fever Cell Colony Lab Source & Virulence Factors Associated Diseases Host Treatment /
related Features Features Characteristics Transmission Defenses & Prevention
infection
Immunity
Brucella g(-) Grows Oxidase (-/+) Animals are reservoir 1. Superoxide dismutase: 1. Brucellosis: “Enteric Ab (IgM, IgG) Treated w/
Species slowly on Cabalase (+) for Brucella species. Allows survival w/in Fever” simlar to Typhoid (a)
coccobacilli blood agar Urease (+/-) Zoonotic infection of PMN & phagocytes Fever. Systemic infection CD4 & CD8 oral doxycycline
TSI (+/-) man occurs by contact involves multiple organs, Tcells, cytokines + intramuscular
aerobic Requires No fermentation w/ infected farm 2. Survives in reticular include GI tract, liver, (RES IFN gamma, & gentamycin
blood media N03 reduction animals such as cows, endothelial cells. B. = reticular epithelial organs. IL-2.
non-motile goats, & pigs. Abortus block Organs á, macrophages Granulomas are or
LPS: associated acidification. (spleen, liver, bone marrow, formed by
antigens, A = Hygiene (handwash!) Intracelluarly releases 5’ lymph nodes, kidneys) delayed (b)
abortus antigen & & inhibits May result in enlargement, hypersensitivity doxycycline +
M = (melitensis) Ingestion of raw milk myeloperoxidase halide endocarditis & p.neumonia. rxn. rifampin
antigen, are system to generate toxic
present Erythritol levels in oxygen. Symptoms: IgG á indicates difficult to treat
placenta (breast, - malaise, chills, sweats, & relapse takes 4-6 weeks
uterus, epididymis) 3. LPS (endotoxin) fatigue, weight loss, non- & it is
productive cough & fever intracellular
4. Serum resistance (intermittent).
Pregnant
2. UNDULANT FEVER/ women should
BANG’S DISEASE / use: Bactrimn +
MALTA FEVER gentamycin
- fever is intermittent or
diurnal. Untreated, results Best to
in chronic flue conditions. pasteurization
May also see bone & joint of milk &
infections, severe vaccination of
depression, & farm animals
osteomyelitis.

Granuloma formation in
liver, spleen, bone marrow
& changes in the organs.

Brucella infections leads to


CHRONIC disease (test
bank: 10 day periodicity)
Infect a variety of animals & can cause disease in humans as opportunistic infections. Under goes UNDULANT FEVER (waves peaks & valleys)

B. abortus (Bang & Malta fever) B. melitensis B. suis B. canis


- Primarily cause disease in cattle - more infectious but primarily in goat & sheep - infections in swine - dogs, foxes, coyotes
- abortion in animals, rich in euorthrodol
Human - mild disease w/ suppurative - severe acute disease w/ complications such as - suppurative, destructive - mild disease. Suppurative
Disease complications uncommon, self-limited granuloma formation, becomes chronic disease w/ granuloma formation complications
Rickettsial- Cell Colony Lab Source & Virulence Factors Associated Diseases Host Treatment /
like agent Features Features Characterist Transmission Defenses & Prevention
Immunity
Coxiella g(-) Grows in Weil-Felix Animals are 1. Superoxide dismutase 1. Q (Query) Fever: acute Igm & IgG Tetracycline for
Burnettii cell culture Reaction reseservoir for - survival within the febrile disease: symptoms useful. pneumonia
coccobacilli or in animals (Rickettsia have Coxiella. Zoonotic acidic phagolysosome include: “flu-like” disease
or in antigen on infection. Infected w/ fever, headache, chest CMI and Doxycycline +
aerobic embryonated surface that farm animals cows, 2. * ACIDOPHILIC pain, chills, malaise, severe cytokines lead TMP-SMZ
eggs / tissue cross react w/ goats, pigs, rabbits, - spore taken up by sweats. VERY to intracellular (Bactrim) w/
related to culture antigen on birds. phagocyte, metabolically INFECTIOUS! 10 bacteria killing of min. 2 years
Rickettsia (needs to certain Proteus activated by acid of may cause illness. Usually Coxiella.
grow w/ species) Transmission occurs phagolysosome self limiting, but may Delayed Type
2 phases growth cells no by aeorsoles (contain (multiplies) become latent infection & Granulomas Hypersensitivity
1. spore/cyst artificial Ox-19 (-) spores) from dried chronic, which may formed by DTH
- Dry phase media Ox-2 (-) urine, feces or milk. 3. LPS (endotoxin): phase reactived (use cortisone). Rxn
- animal urine acceptable, Ox-K (-) variation
2. veget. phase not even w/ Ingestion of raw milk a) phase I: highly inf., 2. Q Fever Pneumonia:
blood) Indirect from infected animal surface antigen blocked Atypical pneumonia w/
Not effective fluorescent b) phase II: less inf. non-productive cough,
spore compare antibody kit on Spores / cysts survives inspiratory crackles & flu-
w/ other smear, at 60C for 1 hr 4. Cysts or “spores” allow like symptoms. No fluid in
endospores detection therefore for Coxiella to survive for lungs
b/c this is g(-) important pasteurization at 62.9C long periods of time
(grown in yolk under adverse conditions 3. Q Fever hepatitis/
** Obligate sacs, look for Test Q Characteristic: Hepatosplenomegaly:
Intracellular! complement - Not vector borne, Fever & observe liver
(can also exist fixing ab, inject difference btw granulomas (upon biopsy),
outside of cell, sputum, blood Rickettsial disease, very rare as result of
doesn’t need into hamster) ticks can be involved in chronic latent infection
to be spread transmitting Coxiella.
by any vector) Ticks are NOT direct 4. Q Fever endocarditis
source. Tick feces on (subacute): culture (-),
cattle hides get associated w/ chronic
transmitted when (latent) infection, valvular
human inhales these heart disease w/ pre-
spores. existing damage of valves.
Incubation months to years.

5. Chronic (latent)
infection: osteomyelitis,
neurological symptoms,
heptatis
Cell Colony Lab Source & Virulence Factors Associated Diseases Host Treatment /
Features Features Charact. Transmission Defenses & Prevention
Immunity
Francisella g(-) Grows on It is Transmitted by direct 1. Lipid capsule: blocks 1. TULAREMIA (RABBIT IgM & IgG are Difficult b/c
Tularensis medium hazardous! method or by vector. Wild phagocytosis & blocks FEVER): ineffective intracellular
coccobacilli containing animals are reservoir. opsonizaiton against
(pleomorphic, blood, 2. Ulceroglandular infection. Antibotics:
very small) glucose, & Zoonotic infection of man 2. LPS (endotoxin) tularemia: contact w/ Granulomas are tetracycline
cysteine occurs by contact w/ infected animal, arthropod formed by DTH gentamycin
facultative (recommend infected wild animals Duch 3. Survive in bite, open ulcer at entry site rxn. T cell streptomycin
anaerobic, chocolate as beaver, rabbit, squirrel & monocytes and PMN (ex. Lower extremity or mediated chloramphenicol
but prefers agar + deer. (protect from ab & trunk), possible bacteremia immunity is penicillin (some)
O2 cysteine) complement). Resistant important to
RABBIT FEVER can be to lysosomal oxidans, 3. Oculoglandular tularemia activate Prevention:
non-motile fatal! include HCl by PMN (eye): conjuctival ulcer, phagocytosis Handling
and inhibits regional lymph nodes, opens for intracellular animals, avoid
Transmissión occurs by phagosome-lysome & drains bloodstream, liver, killing, infected animals
tick, mosquito, or fly fusion spleen, lungs. Invasiness &
vectors. Contaminated goes to the blood stream. Vaccine:
water (ex. Contaminated Live-attenuated
H20) maybe envolved in 4. Pneumonic tularemia: if vaccine used
transmission… also in reach lungs by blood or by only on people
stagnant water. respiratory route (aerosol) at risk animal
then spread by person by handlers
Transmitted via aerosols person. High mortality rate.
Especially inhaled version!
Inoculum size to be ill:
10 bugs tick bites 5. Typhoidal tularemia:
50 bugs if inhale ingest food or water (larger
10^8 ingested inoculum size) GI & fever
symptoms can cause sepsis
w/ multi-organ involvement,
even pneumonia

6. Glandar: painful
adenopathy w/ overlapping
ulcers
Cell Colony Lab Source & Virulence Factors Associated Diseases Treatment /
Features Features Charact. Transmission Prevention
Bartonella g(-) White, rough, Oxidase (-) Cats (blood) are reservoir 1. LPS (endotoxin) 1. Bartonellosis (CAT Self-limiting
henselae mixed w/ tan Catalase (-) SCRATCH FEVER): symptoms
curved rods circular Zoonotic infection of man 2. Type IV pili – adhesion, begin 2 weeks exposed. Local Can scratch
Easy to see w/ occurs by the bite or twitch motility lymphadenopathy (lymph disease treat w/
aerobic Grow slowly Ag scratching of an infected nodes), fever, & pustules at needle aspiration
on selected impregnation cat 3. Facultative intracellular scratch site. It is self-limiting but NO
motile blood agar techniques pathogen last months to years. Forms antibiotics
Man to man transmission granuloma.
twichting 15% CO2 NOT observered Bacillary
motility 2. Bacillary angiomatosis: in angiomatosis &
Facultative Can survive in saliva, immunocompromised patients sepsis treated w/
intracellular inhale lice feces, crushed (AIDS) observe chronic erythromycin
pathogen lice spreading cutaneous & visceral
lesions, Kaposi’s sarcoma-like
Causes conjuctiva infection lesions may appear, differ
histologically.
Bacteremia & sepsis may also
occur in these patients

3. Sub-acute bacterial
endocarditis

4. Bacillary peliosis hepatits


(Peliosis): cyst of blood-filled
granulomas

5. Conjunctiva infection
(Oculoglandular syndrome):
swelling of eye, jaw, cervical
lymph nodes
Cell Colony Lab Source & Virulence Factors Associated Diseases Host Treatment /
Features Features Charact. Transmission Defenses & Prevention
Immunity
Pasteurella g(-) Grows on Oxidase (+) Animals & birds are 1. LPS (endotoxin) Acute cellulitis: symptoms T-cell mediated Penicillin or
multocida blood agar Catalase (+) reservoir, upper include erythrema, swelling, immunity is ampicillin &
coccobacilli produce Indole (+) respiratory tract & saliva 2. Large polysaccharide, & pain at bite / scratch site. important. tetracycline.
w/ bipolar yellow Methyl red (-) are involved in capsule: anti- Usually the host injuries Strong acute
staining colonies w/ Non- transmission. phagocytic occur on leg, arm, or face. inflammatory Allergic patients
“musty” hemolytic response w/ - doxcycline
facultative odor Sucrose Zoonotic infection of man 3. Pili – mediates Associated w/ severe bites! PMN is very
anaerobe utilization (+) occurs by the bite or attachment important Avoid contact
Grows best scratching by infected dog Complications include: w/ wild &
non-motile w/ 02 at or cat. 4. beta lactamase: in - tendonitis domestic
37C some strains - osteomyelitis animals & birds
intracellular Man to man transmission - abscess formation
pathogen not observed
Leads to systemic septicemia
Commensals in some or meningitis
animals
AIDS patient: complication
includes sepsis
Flea Cell Colony Lab Charact. Source & Virulence Factors Associated Diseases Host Treatment /
borne Features Features Transmission Defenses Prevention
Immunity
Yersinia g(-) Grows on Oxidase (-) Animals are the 1. LPS: fatal, septic shock BUBONIC PLAGUE: Granulomas ASAP! w/
pestis blood agar. Catalase (+) reservoir – Zoonotic 2. V and W antigens: exact f(x) w/in 2-3 days of flea-bite. are result of Streptomycin
short Lactose (-) (also Vector-borne unknown but involved in Symptoms include: fever, DTH.
coccobacilli Epicellular- Urease (-) infection) extracellular survival as well as chills, painful Chloramphenicol
w/ bipolar grows both Methyl red (+) intracellular survival & lymphadenitis (buboes). Acute Tetracylcine
staining on tissue & VP (-) Fleas from infected multiplication in macrophage; Inguinal, axillary, femoral inflammatory
on plates animals RODENTS, Resists phagocytic killing & are & cervical lymph nodes response by Hypotension use
facultative Gram satín & rats, & squirrels facultative intracellular involved, & may swell. PMNS. Both dopamine.
anaerobe Grows best Wayson’s satín involved in pathogens. Bacteremia, sepsis CMI and
at 28C w/ to look for transmission of disease 3. Coagulase: blood to clot follows. Vasculitis may sIgA and IgG Disease is
non-motile oxygen bipolar cells to humans. Fatal during flea bloodmeal, lead to gangrene (Black are important prevented by
transmission, & regurgitate into the next bite death, necrosis of against rodent control
facultative Direct immuno- survives for several capillaries, blacken body). infection. but also need to
intracellular fluorescence months in infected 4. Fibronolysin: produced, Infection is fatal. If control rat flea &
pathogen carcasses, sputum, & breaks the clot & spreads swallowed, stools maybe human flea. Use
flea feces. bloody + pus (enteric pesticides.
5. Iron acquisition: inflammation). May last
May cause enteric - takes up iron (hemin) by for weeks w/ treatment.
disease from ingest siderophore-independent
contaminatation. Produce siderophore (only one) Septic plague: NO
occurrence of buboes,
Inhalation of 6. Protein capsule-complex: found in children bites
aerosolized Y. Petsis Fraction1: not produced in rat from flea. Leads to
from infected humans flea – plasmid encoded allows intravascular coagulation
or while handling for survival inside of w/ vascular & renal
infected live animals phagocytes collapse.
carcasses. 7. Promote invasion &
proliferation within host cells Pneumonic plague:
Pneumonic Plaque and resistant to killing by host already in lungs, inhales.
(rare & deadly!) LCR plasmid products are Results in
expressed at 37C and at âCa++ bronchopneumonia.
Sylvatic plaque which directs V and W antigen Symptoms include: fever,
(transmit by fleas on synthesis. cough w/ blood & loaded
rabbits, squirrels vs. 8. Pesticin: bacteriocin, makes w/ bacteria. Deadly &
urban plaque spread by the organism more virulent rarest form. HIGHLY
rats) Flea jumps onto 9. pur protein: allows uptake of INFECTIOUS! 90-100%
human. Then spread adenine & guanine nucleotides death rate.
human to human. 10. pigment production
11. Pathogencity island Sylvatic plague: fleas on
Organism survives in 12. YopE, disrupt actin filament rabbits, wild rats to fleas
animal blood & flea gut 13. YopJ/P: initiate apoptosis on urban rats to human
14. Plasminogen activator fleas to human.
15. Type III: secretion system
Cell Colony Lab Source & Virulence Factors Associated Diseases Host Treatment /
Features Features Charact. Transmission Defenses & Prevention
Immunity
Rickettsia g(-) Grown in Weil-Felix Vertebrate animals are 1. LPS (endotoxin) Highly infectious (but lowest Ab may be Oral
Typhi cell culture Rxn reservori for R. typhi in pathogenicity of the important doxycycline
small (agglutinate particular rats. Zoonotic 2. Factors inducing Rickettsia). 10 organisms to initial barriers (makes teeth
Aka coccobacilli w/ Proteus infection of man occurs by phagocytosis. Inside cause disease! against yellow in
vulgaris) ex: bite of infected flea phagocytes the infection. children)
R. Mooseri non-motile (vector). In infected flea organism survives & 1. Endemic flea-borne Previous
Ox-19 (+) (rat, cat). enters into the host TYPHUS aka Murine typhus infection Preg. Women
Vec./spec aerobic Ox-2 (-) endothelial cells. Evade Systemic inf. following a flea confers long- are treated w/
Flea-ty Ox-K (-) Maybe transmitted by host cell lysis (escape bite. Rickettsemia causes lasting chloramphenicol
end OBLIGATE transovarian method from cell). vasculitis of the capillary immunity.
INTRA- (moma fly to baby fly) beds of many organs Killing rat is
ENDEMIC CELLULAR 3. Produe particularly of the liver & Observed cross- not enough, b/c
PATHOGEN Also flea bites (w/ feces) phospholipase A skin. immunity w/ flea could jump
Radiates transmitted into bite (lecithinase) which R. prowaseckii. to humans.
outward wound. lyses phagosomes Typhus produces fever,
enabling rickettsia to chills, headache, & macular
enter into host cell rash mosly on trunk
cytoplasm. (hallmark) and may
eventually spread outward
4. Organism able to to extremities.
survive in rat blood &
flea feces This differs from Spotted Fever,
rash start on palms & soles &
5. Loosely adherent radiate inward to trunk.
slime layer.
Cell Colony Lab Source & Virulence Factors Associated Diseases Host Treatment /
Louse-Borne Features Features Charact. Transmission Defenses & Prevention
Immunity
Bartonella g(-) Grows very Oxidase (-) Humans were thought to 1. LPS (endotoxin) 1. Bartonellosis – Trench Both IgM and Doxycycline or
Quintana slowly on Catalase (-) be only reservoir BUT 2. Facultative Fever (5 day fever). IgG are chloramphenicol
(Rochalimaea curved rods selective cats too. Indirect intracellular pathogen Remember it is facultative important
Quintana) agar transmission from man intracellular & has Pregnant women
motile medium to man occurs by BITE of periodicity cycle in 5 day - erythromycin
“Trench (twitching) the VECTOR, the body periods. Symptoms include
warfare” louse fever, headache, exhaustion.
aerobic áâ temp. during this
Epidemic periodicity (Quintana),
responsible OBLIGATE chills severe bone pain &
INTRA- transient rash on trunk &
CELLULAR may see splenomegaly &
PATHOGEN myalgia. Self limiting but
may relapse.

2. Reemerged in AIDS
patient & homeless
(immuno-compromised)

Inhalation of louse feces is


another method of
transmission.

Epidemics is common.

Associated w/ filth &


poor santition.
Cell Colony Lab Source & Virulence Factors Associated Diseases Host Treatment /
Louse-Borne Features Features Charact. Transmission Defenses & Prevention
Immunity
Rickettsia g(-) Grown in Weil-Felix Humans are primary 1. LPS (endotoxin) 1. Epidemic louse-borne Ab may be Oral doxycycline
prowaseckii cell culture Rxn animal reservoir but typhus:Systemic f(x) important
very small (agglutinate other animals can also be 2. Produce factors following a louse bite. initial barriers Pregnant
coccobacilli w/ Proteus reservoir. Flying inducing Rickettsia cause vasculitis of against women are
Mnemonic vulgaris) ex: squirrels. Zoonotic phagocytosis. Inside capillary beds of many infection treated w/
Vect/Spec/dis non-motile infection of man occurs phagocytes the organs particularly of the chloramphenicol
Lousy pr-epi Ox-19 (+) by bite of infected louse organism survives and liver & the skin. Previous
aerobic Ox-2 (-) (vector). Arthropod enters into the host infection
Epidemic Ox-K (-) likely to be squirrel flea. endothelial cells. Clotting abnormalities confers long-
responsible OBLIGATE Evade host cell lysis. lasting
INTRA- Symptoms include: immunity
Spreads CELLULAR 3. Produces - high fever, chills, heaches,
outward PATHOGEN phospholipase A & pain
(lecithinase) which
lyses phagosomes Rash mostly on trunk (key
enabling rickettsia to feature!) & spreads
enter into host cell outward toward extremities
cytoplasm. EXCEPT face, soles &
palms. (differs from
4. Organism is able to Spotted Fever – which starts
survive in rat blood & from palms & soles &
flea feces radiate inwards to trunk)
Infected louse may be
transmitted to man via 5. Slime layer See neurological changes
inhalation of dried lice include delirium & stupor.
feces or by bite wound or
crushed into the skin. More life threatening: During cold weather may
Bartonella Quintana lead to gangrene of feet &
NOT passed transovarian fingers.

Unlike other rickettsia 2. Brill-Zinsser Disease:


diseases: can also see Reactivated epidemic
man to vector to man typhus (BuZy Pissing) (less
transmission. Still needs severe than primary
the vector as source! infection) May occur years
after initial disease.
Chronic carrier w/ less
intense rashes b/c of partial
immunity.

3. Copmlications: CNS
dysf(x), myocarditis
Cell Colony Lab Source & Virulence Factors Associated Diseases Host Treatment /
Tick-Borne Features Features Charact. Transmission Defenses & Prevention
Immunity
Rickettsia g(-) Grown in Weil-Felix Animals (wild mammals 1. LPS (endotoxin) 1. Rocky Mountain Spotted Ab may be Oral do Oral
rickettsii cell culture Rxn & birds) & hard ticks Fever (RMSF): Systemic inf. important doxycycline
very small (agglutinate 2. Produce factors following a tick bite (tick initial barriers
coccobacilli w/ Proteus Zoonotic infection of inducing saliva). Rickettsemia against inf. Pregnant
vulgaris) ex: man occurs by bite of phagocytosis. Inside causing vasculitis of the Previous inf. women are
Mnemonic non-motile infected tick (vector). phagocytes the capillary beds of many confers long- treated w/
vect/spec/dis Ox-19 (+) organism survives & organs particularly of the lasting chloramphenicol
Ricky-ticky aerobic Ox-2 (+) Infected tick, organism enters into the host lungs & skin producing a immunity.
spot Ox-K (-) may be transmitted to endothelial cells. fulminant disease. Prompt removal
OBLIGATE tick offspring by Evade host cell lysis. of ticks also
Caus. Agent: INTRA- transovarian method. Symptoms: important as
Rocky Mount. CELLULAR 3. Produces - rapid onset, high fever, preventive
Spotted Fever PATHOGEN phospholipase A nausea, vomiting, myalgia, measure.
(RMSF) (lecithinase) which headache see macular rash
#1 rickettsia lyses phagosomes
disease & non- enabling rickettsia to Rash spreads from
vector borne enter into host cell extremities to trunk. (rash
disease in US cytoplasm (also works inward)
pop. deacidify vacuole)
Disease may lead to
Also note: Has to be attached 24-48 4. Organism is able to pulmonary failure, renal
Tick’s may hr in order to become survive in rat blood & failure, encephalitis, coma,
deliver encouraged w/ blood & flea feces & death.
Rickettsia or to transmit the organism
lyme disese to man. 5. Slime layer

Test Question: #1 non-vector borne disease in US Rocky Mountain Spotted Fever – primarily found on East coast
Test Question: #1 vector borne disease in US Lyme disease (carried by tick)
Spirochetes (cork screw) Chlamydia
- outer most layer is referred to as an outer sheath (glycosaminoglycan) - Gram (-), obligate intracellular organisms w/ unusual “life cycle”
- underneath is the outer membrane (peptidoglycan, lipids + covalently linked - NOT spirochetes!!!
proteins) - Obligate intracellular parasites !!! Can not synthesize ATP!
- encase the periplasmic space where endoflagellae are located o No cytochromes or flavoproteins. Can’t metabolize glucose to pyruvic
- directly underneath the periplasmic space is inner membrane (cytoplasm) acid using pentose phosphate pathway
- Cytoplasmic tubules (body fibrils)
- NO LPS Chlamydia Life Cycle:
- Has a layer of peptidoglycan 1. Small dense cell: ELEMENTARY BODY (infectious form but NOT replicating)
it is outside of the host. Enters the host via phagocytosis DNA:RNA, 1:1 ratio =
Test Question: Endoflagella (axial filaments) inside of outer membrane NOT exposed to infectious particle
outside but can be shed in infected individual. Lives in intestines & in mouth 2. Next 8 hr, elementary body reorganizes into large, less dense cell called
reticulate (or initial) body
3. Reticulate body: (this replicates) , grows in size & divides by binary fission
(DNA:RNA ratio is 1:4) this is NON-INFECTIOUS form
4. 24-48 hr, reticulate bodies reorganize into dense elementary bodies &
developmental cycle is complete when host cell liberates the small dense
infectious cells.
Serology
a) Non-specific blood tests: (agglutination) indicate present infection, since
individual become (+) upon infection but levels decrease during chronic disease.

Tests for Syphilis


1. Complement Fixation (Wasserman) indirect test: used to detect presence of
reagin antibodies (reagan is antibodies directed against cardiolipin)
Cardiolipin is an antigen which is released by damage host cells & present on
the treponeme.
2. Rapid Plasma Reagin (RPR): also a reagent cardiolipin & is a carbon
agglutination test
3. Venereal Disease Research Lab (VDRL) test: Cardiolipin + serum, check for
clumping (means + results of antibodies against cardiolipin) AKA Flocculation
test
4. Treponemal Specific Blood tests
5. Fluorescent treponemal antibody Absorption test (FTA-abs)
6. T. pallidum Hemagglutinin Assay (TPHA)

Vocabulary Disease Symptoms:


1. Charcot’s disease: trophic degeneration of knee joint. Loss sensation due to
neurosyphilus
2. Condyloma: fusion of 2nd syphilitic lesions around the anal areas
3. Gummata: ulcerous skin lesion (chewy gum)
4. Hutchinson’s teeth : teeth are translucent. Incisor-like
5. Saber shins: affects long bones
6. Saddlenose: perforated nasal septum
7. Tabes dorsalis: destruction of dorsal roots of spinal cord
Cell Colony Lab Source & Virulence Associated Diseases Host Treatment /
Features Features Charact. Transmission Factors Defenses Prevention
Immunity
Treponema Thin (0.5 u) Can not be Look for Man is only reservoir. 1. Glycolipid Syphilis (3rd most common STD) IgG and IgM Syphilis is
pallidum tightly coiled cultured in spirochetes instead of LPS develops by treated w/
spirals (5-15u) vitro!!! using direct Horizontal transmission (Glycolipid is rich 1. Primary syphilis: systemic inf end of 2nd penicillin (at all
miscroscopy by sexual contact. in cardiolipin & following sexual contact. stage are stages) &
g(-) difficult to Grows in other Typically hard, painless chancre important. penicillin can
see b/c too thin rabbit Darkfield & Vertical transmission phospholipids to forms site of entry (genitals, mouth Immunity also be given
testicles or immuno- (mother to child) through test for syphillus) or anus) w/ spirochemia takes a long prophylactically.
Visualized w/ in rabbit fluorescent the placenta in utero. (spirochetes in blood stream) time to
darkfield epithelial methods & 2. Motility Sympt. 4-6 weeks. Most develop in If sensitive Pen.
microscopy! or cell culture SEROLOGY Highly infectious! Less contagious stage. May see > 1 part b/c so Use ==>
immuno- than 10 spirochetes to 3. complex outer chancre in AIDS patient. there are few erythromycin or
fluorescent cause disease! membrane proteins on tetracycline
microscopy & structure w/ few 2. Secondary syphilis: persistent surface of T.
silver membrane bound spirochemia, 3 weeks – 3 months. palladium.
impregnation outer proteins Papular rash develops entire
body. Spreads from trunk to soles, Little
Moves using 4. Outer membrane genitalia, & mucous surfaces. protection
axial filaments, promotes AKA Great Pox (Great Imitation) immunity
ENDOflagellae adherence to host w/ fever & soar throat. Spirochete during 1◦
cells by “tips” may invade any organ of body. and 2◦ stages.
Microaerophilic bacteria Rash may last up to many months. T cells are
Contagious. Organism may be involved in
Sensitive to 5. Hyaluronidase: isolated from rash. May develop gumma f(x)
environmental enables spirochete LATENT disease. Condylomata
influences to invade host (bacteria continue to multiply &
tissue deep-seated lesions)

6. Survive 3. Tertiary syphilis (LUES)


intracellularly NOT CONTAGIOUS! Disease
(phagocyctic cells) affect any organ, may re-occur. T.
& extracellularly in pallidum, only observed in the
host CSF. Lesions on tongue, gummata
lesions, & other effects on skin,
7. fibronectin (anti- bones, & blindness & Charcot’s
phagocytic) blocks knee
phagocytosis
4. Congenital syphilis:
transplacental inf. of fetus at 1◦2◦
latent stages in mother. Death of
fetus (blindness or deafness). Also:
saddle nose, saber shins, skin rash,
Hutchinson’s teeth, heart
malformation, & liver damage.
Cell Colony Lab Source & Virulence Associated Diseases Host Treatment /
Features Features Charact. Transmission Factors Defenses Prevention
Immunity
Leptospira g(-) spirochete Can be Serological Parasites of non-human 1. LPS (endotoxin) Severe case: (effect liver & kidney) IgM & IgG Oral
Interrogans cultured in tests such as animals (ie livestock & LEPTOSPIROSIS which is usually are doxycycline,
thin vitro. agglutinating dogs). Best friend (dog 2. Hyaluronidase a biphasic disease. Moves quickly important erythromycin,
complement prone to infect man) - invasiveness & effects blood. Autoimmune or penicillin
tighly coiled Grows fixation or response
spirals w/ rapidly in lysis in Animal kidneys are 3. Outer surface a) Leptospiremic phase: presence may be
unusual complex presence of resvoir. Eating kidney, of leptospires in blood & CSF. involved in
hooked ends media complement. or consuming water 4. Maybe hemolysin Abrupt onset of fever, shaking disease
contaminated w/ animal chills, headache, & myalgia. process
Use darkfield Uses fatty Culture from urine. 5. Motility &
microscopy, acids & blood, CSF, & thinness (penetrate b) Immune phase: Accompanied
immuno- alcohols as urine. Zoonotic infection intact mucous by IgM production. Aseptic
fluorescent carbon membranes; skin meningitis (no bacteria in blood
miscroscopy or source. Survives for months in via small cuts or but effects brain) . Onset of
stained by soil & water but sensitive abrasions) hepatic & renal failure &
Silver Likes to Chlorine widespread hemorrhages & shock.
impregnation alkaline Also known as Weil’s Disease due
technique. conditions. to an autoimmune response
causing liver damage b/c of
Obligate aerobe molecular mimicry. AKA
INFECTIOUS JAUNDICE
Motility (via 2 (yellow/green liver) .
endoflagellae
but least motile Has 2 phases:
of all 1. Anicteric phase: w/o liver
spirochetes) 2. Icteric phase: w/ liver involve

Interrogans:
shape, question
marks ???
complex outer
membrane
__________ structure
* Twitch contains LPS
motility
differs from
spirochetes
Lyme Cell Colony Lab Source & Virulence Associated Diseases Host Treatment /
disease Features Features Charact. Transmission Factors Defenses Prevention
Immunity
Borrelia Thin Cultured in Darkfield Primary reservoir is the 1. Glycolipid LYME DISEASE: > 1 stage IgG and IgM Stage 1:
burgdorferi vitro microscopy mouse including white (similar to against outer Oral doxycycline
Loosely coiled food mouse, dusty footed Treponema) NO Stage 1: appears surface
long spirals Grown on Use ELISA wood rat, chipmunks, LPS - Erythema Migrans (EM) or antigens Stage 2 & 3:
#1 vector (spirochete) Barbour- and DEER. Erythema Chronicum Migrans: (Osps) IV ceftriaxone
borne Stoenner- Western blot 2. Surface protein: Lesion or rash (Bull’s eye lesion) important
disease US g (-) Kelly also PCR Vectors are the primarily produces many - Symptoms: but undergo
medium Ixodid ticks different surface headache, fever, stiffneck, malaise, antigenic Prevention Tips
Use darkfield contains antigens during & lymphadenopathy. variation. Dress properly
microscopy, bovine infection. w/o skin
fluorescent serum ANTIGENIC Stage 2: exposed, use
miscroscopy + albumin VARIATION - Lyme arthritis (uniarticular ie insect repellent
acridine (BSA) & involving 1 joint) Reversible w/
organe dye or rabbit 3. penetrate treatment.
Infection of man occurs
stained by serum at epithelial cells - Symptoms: neuroborreliosis
by tick bite & transfer of
Silver 30C (meningitis, peripheral
organism during tick-
impregnation 4. Survives in blood neuropathy, encephalitis, Bell’s
engorgement!
technique. & in tick – ability to Palsy) may last for months – years
Transmitted in vector by
transovarian route. Also resist phagocytosis
Microaerophilic Stage 3:
tick saliva or deposition
5. Hemolysin Follow a latent & asymptomatic
of feces into wound.
Rotational period. Late or Chronic Lyme
motility disease.
(endoflagellae, Symptoms: similar to stage 2 but
axial filaments) worse. Mostly arthritis.

DNA is
fragmented w/
some being
linear
Cell Colony Lab Source & Virulence Associated Diseases Host Treatment /
Features Features Charact. Transmission Factors Defenses Prevention
Immunity
Borrelia Thin Cultured in Darkfield Humans are believed to 1. Glycolipid Relapsing Fever: > 1 stage IgM & IgG Oral doxycycline
recurrentis vitro microscopy be only reservoir against outer Penicillin
Loosely coiled (epidemic form of 2. Surface protein Stage 1: surface Tetracycline
Grows disease). Chloramphenicol
long spirals (VARIABLE 1st fever symptoms (maybe fatal antigens are
“relapse (spirochete) slowly on MAJOR due to shock-hypotension). important Erythromycin
fever” complex at Vectors for the epidemic PROTEIN) which Spirochetes from lice (or tick) leads to (children &
g (-) 30C organism are lice produces many penetrate the skin & skin & spread complement- pregnant women)
Epidemic (therefore, louse-borne different surface through the blood. Treat w/ dependent
Use darkfield disease) antigens, resulting antibodies will clear most lysis.
microscopy, in several relapses circulating organisms, so host
fluorescent Endemic form of disease (ANTIGENIC experience a non-febrile latency Cytokines
miscroscopy + is found in animal & VARIATION). period 7-10 days. may be
acridine human reservoirs & involved in
organe dye or appear to be spread by 3. ability to Stage 2: disease
stained by ticks. penetrate epithelial Spirochetes go through antigenic resolution
Silver cells variation & a new variant will be
impregnation produced. Results in bacteremia &
technique. 4. Survives in blood a second fever stage (relapse).
& in lice – has the
Microaerophilic ability to resist
phagocytosis
Rotational
motility
(endoflagellae,
axial filaments)

DNA is
fragmented w/
some being
linear
NOT Cell Colony Lab Source & Virulence Associated Diseases Host Treatment /
spirochetes! Features Features Charact. Transmission Factors Defenses Prevention
Immunity
Chlamydia g (-) does not Grows in Difficult to Humans are reservoir. 1. Intracellular Male STD: most common STD in Ab not Oral doxycycline
trachomatis have typical cell culture diagnose. survival US. Urethritis involve urethral effective. Sulfonamide
cell wall of Infection of man occurs a) stimulates non- discharge. (SYPMTOMATIC) (unique)
peptidoglycan LGV test by sexual contact & by phagocytic PMN
but sensitive to dead bacteria autoinoculation to other epithelial cycle to Female STD: Cervicitis, effective in Erythromycin
Penicillin injected parts of body. engulf them endometritis, PID involve killing.
Most under skin & discharge. May cause sterility Cefoxitin &
common compact look for DTH. Mother to child b)Inhibit women are ASYPMTOMATIC doxycycline if
STD inclusion body transmission is possible phagosome- concurrent w/
w/ glycogen, Serology at birth. lysosome fusion Swimming Pool Conjunctivitis: N. gonorrhea
make folic acid 15-20 (all 3 chlamydia Spread by fingers, fomites & even infection
therefore different Maybe transmitted by have them!) chlorinated water into eye.
sensitive to serotypes direct contact with ADULT INCLUSION Antibiotics such
sulfonamide fingers, contaminated 2. Unusual life CONJUCTIVITIS: STD as erythromycin
ONLY!!! towels & clothing. cycle: elementary or tetracycline
(differs from . bodies can infect TRACHOMA: are used in form
C. psittachi) Can be transmitted host macrophages Chronic eye inflammation by of eyedrops for
sexually to other adults & epithelial cells, infecting eye. new borns.
leading to genital becomes reticulate
symptoms. Can be bodies & produce Lymphogranuloma venerum:,
spread by fingers, new elementary (LGV): develop into buboes and
towels, clothing… can bodies ==> lead to elephantiasis of genitalia &
even spread in released when host rectal structures as result of
chlorinated water cell lyses. perirectal scarring.
(swimming pool
conjunctivitis) 3. Endotoxin Neonatal infection:
Due to vaginal passage can cause
4. “toxic factor” disease by either being aspirated
into lung or by conjuctiva of
5. Type III Secretion neonate.
system a) Infant atypical pneumonia:
- long term complications & otitis
media
b)Inclusion conjunctivitis:
may have similar to trachoma

Reiter’s Syndrome:
Arthritis, periostitis on calcaneus
(heel) pain AKA Lover’s Heel
NOT Cell Colony Source & Virulence Factors Associated Diseases Host Treatment /
spirochetes! Features Features Transmission Defenses Prevention
Immunity
Chlamydia g (-) does not Grows in Contact w/ parrots & 1. Intracellular survival: Psittacosis (ornithosis) Ab not Doxycycline and
psittaci have typical cell culture other psittacine birds can a) stimulates non- PARROT FEVER: effective. azithromycin
(psittaci) cell wall of spread to other animals phagocytic epithelial cycle Can exist as asymptomatic,
peptidoglycan to engulf them Alternatives:
(cattle, swine, cats, dogs) likely to be severe, PMN very
Ofloxacin,
Parrot but sensitive to frequently fatal effective in erythromycin,
Fever Penicillin Can get into: b) inhibit phagosome- pneumonia. Can involve killing. amoxicillin
environment from feces lysosome fusion other organs: jaundice, (pregnant women)
diffuse & survive a long time ! (all 3 chlamydia have acute thyroiditis, Previous
inclusion body them!) meningitis w/ delirium. It infection Quarantine
is a latent infection. Lung provides no imported birds
no glycogen 2. Unusual life cycle: inhaled form it is atypical immunity &
Elementary bodies can pneumonia. Spread by subsequently
can’t synthesize infect host macrophages & human to human. see relapse &
folic acid, epithelial cells reinfection.
resistant to
sulfonamide! 3. Endotoxin similar to LPS

4. Toxic factor

5. Type III secretion system


NOT Cell Colony Source & Virulence Factors Associated Diseases Host Treatment /
spirochetes! Features Features Transmission Defenses Prevention
Immunity
Chlamydia g (-) does not Grows in Humans are reservoir. 1. Intracellular survival: 1. Atypical pneumonia: Ab not Doxycycline and
pneumoniae have typical cell culture Occurs in respiratory a) stimulates non- (sneezing or coughing): mild effective. azithromycin
cell wall of droplets. phagocytic epithelial or asymptomatic but in
peptidoglycan cycle to engulf them Alternatives:
elderly, it can be fatal. PMN very
Ofloxacin,
but sensitive to Epidemics are possible Involves in the lower lobe. effective in erythromycin,
Penicillin TWAR strain caused b) inhibit phagosome- Pneumonia and bronchitis in killing. amoxicillin
acute respiratory disease. lysosome fusion (all 3 adolescents & adults is often (pregnant women)
diffuse chlamydia have them!) seen w/ pharyngitis and Previous
inclusion body sinusitis. infection
Other Symptoms: non- provides no
no glycogen 2. Unusual life cycle: productive cough, fever, immunity &
Elementary bodies can chills, headache & malaise. subsequently
can’t synthesize infect host macrophages Complication of coronary see relapse &
folic acid, & epithelial cells artery disease & reinfection.
resistant to endocarditis.
sulfonamide! 3. Endotoxin similar to
LPS

4. Type III secretion


system
Mycology (Fungi) Zygomycetes: (phycomycetes):
- ex. bread molds, most primitive, filaments, aseptate, sexual & asexual
Characteristics: (sporangium formation)
- Eukaryotic cells w/ cell walls (no peptigylcan)
o Chitin, cellulose, glucans, mannans (ex. polymers of sugars) Ascomycetes (Ascomycotmia):
o Non-photosynthetic, Heterotrophs, saprophytes, or saprobes, parasites Asexual Sexual
- Cell membrane - conidia - ascospores (incl. dermatophytes) and spherules
o Ergosterol
o No motility Basidiomycetes:
- Cryptocococucus neoformans (only one with capsule), anti-phagocytic! - septate hyphae, sexual spores

2 forms: Deuteromycetes (Fungi Imperfectii)


a) unicellular yeast (chains) - has sexual stages
b) multicellular mold (septate) (filaments structures aka hyphae)
Culture Fungi:
- Coenocytic: several nuclei present in cytoplasm of 1 cell (multinucleated) - Sabouraud agar
- Mat of hyphae = mycelium or thallus o Slightly lower pH (more acidic)
- Vegetative hyphae = penetrates substrates for nutrient acquestion o Contains dextrose & peptone
- Reproductive hyphae: aerial structures for reproduction o Chloramphenicol to inhibit bacteria
o Cyclohexamide (Acti-Dione) inhibit saprophytes
c)Dimorphism triggers: - Molds – grows away from plate, looks fuzzy
1) 25C ==> forms macrocondia (mold), Chlamydia spore o Vegetative hyphae = “roots”
2) 37C ==> budding yeast form a non-mold form (C. immitis) - Yeasts – creamy opaque colonies

d)Reproduction: Dermatophyte Test Medium (DTM)


Anamorphic Telomorphic - inhibits bacteria
- cyclohexamide to inhibit saprophytic molds
- Asexual - Sexual
- dermatophytes turn medium yellow ==> red

- Sexual form NOT identified: (Fungi Imperfectii or deuteromycetes) Corn-meal Agar


- 99% of ASEXUAL spore - test for spore formation
o Fragmentation - visualize chlyamydospore & blastocondia
o Fission (equal size) - production candida albicans
o Budding (new buds, vary in size)
o Spore formation: mostly budding Monomorphic fungi
o Yeasts: fission, budding - dermatophytes
- Spores produced in sac sporangium w/ spores sporangiospores o E (NS)
o M (SH)
Thallospores: o T (NSH)
- arthrospores: barrel shaped, thick-walled produced by hyphae fragmentation - Cryptoccous neoformans (encapsulated, endosporulating, spherules)
- chlamydospores: swelling of hyphal fragment, thick double wall - Aspergillus fumigatus (fungus ball)
- blastospores: simple budding from parent cells
Dimorphic fungi
Sexual Reproduction (cells are 2N), “Perfect state” - Histoplasma (dusty environment, bird feces)
- oopspores - blastomyces (valley fever)
- zygospores - coccidiodies
- ascospores (sac) - sporothrix (rose garden cuts)
- basidiospores: club shaped (mushroom)
Lab diagnosis: Tinea Corporis (non-hairy parts of body)
- Potassium hydroxide: (KOH) used for diagnosis + heat for ID - mistaken for lupus
- Calcofluor: fluroscent stain, UV microscope - scaly
- Gomori methenamine silver stain (GMS): tissue stains red & fungus blue - popular eruptions
- Lactophenol cotton blue dye: (stain hyphae): aniline stains fungal
- India ink & test for capsule (-) stain ==> cryptoccocus neoformans Tinea Pedis (Athlete’s foot)
- public shower’s, swimming pools, watery lesions btw toes, show peeling &
3 types of transmission: cracking of skin
- Anthropophilic ==> human to human - caused by: Trichophyton rubrum & T. mentagrophytes
- Zoophilic ==> animal to human, ex. Microsporum canis
- Geophilic ==> soil to human, ex. microsporum gypseum Tinea Favosa (form of scalp)
- yellow, cap-shaped crusts
Candida – pseudohyphae, burrows into tissue - looks like a honey comb
- yeast disseminate easier body via blood - T. Schoenleinii (fluoresces yellow-green)

Antifungal agents: (ex. Nystatin, Amphotericin B) Tinea Unguinum (nails - infection)


• Polyenes: creates large pores in fungal cell membrane by binding to erogsterol - superificial, only pits of nails
• Azoles: ex. ketoconaxole, fluconaxole, itraconazole, inhibit ergosterol, block - involvement of nail surface w/ invasion beneath nail plate
cytochrome P450 enzyme - onychomycosis (nail infection) caused by non-Dermatophytic
• Allylamine: ex. naftine, binafine, terbinafine, lamisil o saprobe fungi ex. Aspergillas
o tolinaftate : ex. tinacitin ==> ergosterol - “Fungal finger nails” ex. car wash workers, common cause Candida Albicans
• Griseaans : inhibit microtubule so cells do not divide
• Potassium iodide (KI) : antifungal therapy to treat sporothrix schenckii infection Tinea Cruris (Jock Itch)
- can be epidemic form w/ exchange of clothing, towels, scaling lesions, itchy &
Mycotic Infection: often dry
- cutaneous
- superificial Dermaphytids
- subcutaneous - not grown on Sabouraud
- systemic - allergy response w/ dermatophyte infections
- IDs (severe itching especially after antifungal agent)
CUTANEOUS INFECTION (Dermatophyte infection): - Fungal organism NOT present
Dermatophytes:
- invade keratinized & cutaneous areas Epidermophyton (NS):
- produce keratinase, collagenase, elastase - thermally monomorphic, no microconidia but macroconidia, cigar shape
- affect stratum corneum (skin) Microsporum (SH)
- grown best at 25C - thermally monomorphic
- not invasive Trichophyton (NSH)
- thermally monomorphic
Tineas (ringworm)
- Tinea Capitis (scalp) itch on scalp w/ 3 patterns
Caused by microsporum species (M. canis, use Wood’s light)
o Endothrix: inside hair shaft
o Ecotothrix: outside hair shaft
o Favic: hyphae
§ arthoconidia & air bubbles form inside hair
§ Honey comb pattern
Trichophyton (NSH) Malassezia furfur & M. Ovalis (Pityrosporum ovale)
Mentagrophytes Rubrum Tonsurans - Dimorphic & long hyphae
Thermal morphic Yes Yes Yes - Comes from furfaraceous or scaly
moid Cell feature - yeast like cells
Cell features Septate hyphae, Septate hyphae, Various size & - hyphae are usually absent
spiral, oval, tear-drop, shape - “spaghetti & meatballs”
microconidia, peg-shaped, cigar microconidia w/ Dxn
grape-like cluster shaped large spherical Disease - Superficial inf. Of skin (Tinea Versicolor, Pityriassis Versicolor)
conidia - skin-inf. non-itchy, scaly, pale spots on shoulders & back, chest, &
Diagnosis Bright RED* upper arms!
Disease Tinea Capitis Black dot Treatment Wood’s Light, orange-red fluorescent hyphae
Tinea Corporis Black spores lead
Tinea Cruris to tinea capitis Piedraia Hortae
Tinea Pedis - dark/black hard nodule hair shaft (Black Piedra)
Tinea Ungium Disease - Base of the hair to fall out
Onychomycosis
Dxn Scraping of Trichosporon Beigelii (T. aschii, T. mucoides, T. inkin)
infected skin, nail, - baige associated w/ white Piedra
KOH Disease - Superificial infection of hair roots

Microscporum Gypseum Complex (M. Canis & M. Audouinii)


- thermally monomorphic mold SUBCUTANEOUS INFECTION (Dimorphic)
Cell feature - Septate hyphae - stays localized
- microconidi, along the hyphae & club shaped
Dxn - KOH, non-treated test w/ sabouraud agar for microsporum canis Sporothrix Schenckii (Gardener’s w/ hay, rose bushes, ferns) puncture of skin
use wood’s lamp (yellow-green) fluorescence - thermally dimorphic yeast-like fungus dematiaceous
Disease - Tinea capitis - “daisy clusters”
- Tinea Cruris - Cigar shaped budding yeast (37C)
- Tinea Pedis - Wrinkled round brown, black colonies
Transmission - dog ==> human (zoophilic) & geophilic transmisión - Infectious occurs (scratches, punctures) during gardening from wood, splinters,
Treatment - affected skin, kept cool & dry - Prevelant in South US
- apply topicl antifungal agents (cream) - Zoonotic transmission: armadillo & infected cats
- tinea capitis ==> oral griseofulvin Disease - painless nodules forming along lymphatic channels
- Lamisil Treatment Potassium iodide

Madurella Spores
Hortaea Wereckii = Exophiala Werneckii : - thermally monomorphic molds, dematiaceous
- Dematiaceous fungi or pigmented fungi - true fungal infection, brown-colored
o Non-invasive Disease - fungal mycetoma (Eumycetoma)
o Pigmented (b/c melanin production) - madura foot, resembles nocardiasis (actinomycetoma)
- Disease: Treatment - difficult to cure
o Produce superificial inf. of skin, Tinea Nigra (black growth on skin)
but non-invasive
o Skin infections, non-scaly, smooth, brown-black painless spots
SYSTEMIC INFECTIONS: (Inhales into lungs) OPPORTUNISTIC INFECTIONS: (Colorized, immunocompromised state of host)
- ALL thermally dimorphic
- True, frank, primary pathogens Aspergillus fumigatus:
- Via respiratory rate (inhaled spores) - thermally monomorphic mold
- inhale spores immunocompromised patients
Blastomyces dermatitidis: - Fungus ball (aspergilloma) spore germinate (Farmer’s Lung) HAY FEVER!!!
- yeast like fungus, looks like figure 8’s - A. Flavus produces aflatoxin potent liver carcinogen. INVASIVE!!!
- multi-nucleated, forms granules of skin
- no cell wall, bird droppings Candida Albicans:
- Mississippi river to East coast - Dimorphic but NOT thermally but grown on corn-meal produce pseudophyae
Dxn - multi-nucleated budding yeasts, single broad-based 888’s - Use as diagnositic: Germ tubes test (true hyphae) this is a unique feature!
Disease - NAmerica blastomycosis (mild pulmonary inf), skin lesion - Part of normal flora (FurFur & tricosporum???)
- Formation of granulomas of skin & bone - Produces ==> chlamydospores are unique to this candida
- cream-colored pasty colonies
Paracoccidiodes brasiliensis: Transmission - normal commensals of human & animal GI, resp, tracts skin &
- thermally dimorphic yeast-like fungus female genital tracts
- UNINUCLEATE!, multiple budding mother cell w/ buds having narrow bones Disease ORAL THRUSH: (AIDS immunocompromised)
- Cause systemic infection, more mucosal surface - produce white patches in mouth & tongue continue to GI tract
- Found mostly in S. America blastomycosis (granulomatous spreads to lungs to other Cutaneous candidiasis: diper rash
mucous surfaces of nose, mouth, & GI tract) INTERTRIGINAS CANDIASIS (Intertrigo):
-Looks like ship’s wheel, steering wheel, pilot wheel - appendages immersed in H20 long periods, moist parts in diabetes
- endemic ONYCHOMYCOSIS: nail inection
VAGINITIS: (Moniliasis): vaginal infection of women
Coccidioides Immitis (C.Posadasii) Chronic Mucocutaneous Infection:
- dimorphic fungus: changes form to spherule at at 37C - CMI failure (T-cell deficiencies) infection associated w/ skin, oral
Cell feature - terminal arthrospores attached directly to hyphae mucosa, respiratory mucosa, GI & GU mucosal surfaces
- spherules in tissue specimens, barrel shapped! Treatment - antibiotics
Transmission - direct inhalation (mold spores, arthrospores) from soil &
dry climates Southwest US (Central or S. America) Cryptococcus Neoformans:
- likes Nitrogen associated w/ bird droppings (feces) - capsule, visible w/ India ink
Disease - Coccidiomycosis, highly infectious disease flue-like - thermally monomorphic yeast
pulmonary infection, CNS (Meningitis)!!! - capsule is anti-phagocytic
- Leads to erythema nodosum (DTH response, red tender Cell feature - oval shapped
nodule on skin) Dxn - Mayer’s stain to see PINK capsules
- Desert rheumatism “San Joaquin Valley Fever”Cali - CSF sample prep. w/ 10% KOH & India ink
- ID test looks for DTH response Disease - result in cryptococcosis (pulmonary infection)
- to meninges & brain thrives on CSF (Meningitis)
Histoplamsa Capsulatum Var. Capsulatum: - occurs often immunocompromised
- dimorphic yeast-like fungus & small oval microconidia - can be unrecognized or asymptomtic
- spiny macroconidia attatched to hyphae, knobby (tuberculate macronidia) Transmission - Via inhalation of yeast found in soil & pigeon droppings
Transmission - direct inhalation of small molds (microconidia) from soil or bird Treatment - Ampotericin B & Latex agglutination!
droppings. They are spiny & knobby looking!
- Mississippi + endemic, Ohio River, S. America, & Africa Mucormycosis (Phycomycosis or Zygomcosis): mucor, absidia, rhizopus species
Disease - Most common respiratory mycotic inf pneumonia, granuloma - invades tissues, brain (nasally) it is a bright mold!
- reticuloendothelial system (RES) found intracellularly w/in Cell feature - aseptate or coenocytic & invasive
macrophages. Like at TB test can use DTH - member of Zygomycetes (phycomycetes)
- Chronic disease: reactivation enlarged granuloma, liver, spleen… - sporangia form at 90 degree angle from hyphae
INDI Organisms # 4

Virus
Size & Structure
• 20-300 nm size (picornavirus ==> poxvirus) appears spheres or rods
• Contain either DNA or RNA, NOT both!
• All have a protein coat, capsid (repeating capsomers)
o Capsid could is outer surface
o Others could have lipoprotein envelope composed of phospholipids bilayer
§ Enveloped: picked up from nucleus or p.membrane or created from cytoplasm ether sensitive
• Composed of nucleic acid & capsid protein is called nucleocapsid
• Shape: spherical (icosahedral) or helical symmetry
o Exception: poxvirus looks like a brick shaped (complex)
• All human viruses have helical nucleocapsid are enveloped
• No naked helical viruses infect humans.
• Icosahedral nucleocapsid can be either enveloped or naked

Viral Nucleic Acids


• Retroviruses: have two copies of genome (diploid)
• DNA Viruses
o Are dsDNA except parvovirus
o Are icosahedral, except poxvirus (brick, complex shaped)
o Replicate their DNA in nucleus, except poxvirus

Virus Family DNA type Virion Envelope DNA replicates in: Major viruses
(associated polymerase)
Parvovirus ssDNA No Naked Nucleus B-19
Papovirus dsDNA No Naked Nucleus Papilloma
circular Polyoma
Adenovirus dsDNA No Naked Nucleus Adenoviruses
linear
Herpes virus dsDNA No Enveloped Nuclues; virus HSV: Varicella-
linear (nuclear) assemble in nucleus Zoster, EBV

Poxvirus dsDNA Yes Enveloped Cytoplasm Variola


linear (DNA dept RNA Vaccinia
polyermase) Molluscum
Contagiosum
Hepadnavirus Partially Yes Enveloped Nucleus, RNA Hep. B
dsDNA intermediate
circular
Mnemonic: Parva’s Papa Adds Her Poxes to Hepas
• (+)RNA Viruses

Virus Family RNA structure Virion Envelope Shape Multiplies Major viruses
(associated polymerase)
Calicivirus ss(+) RNA No polymerase Naked icosahedral cytoplasm Norwalk agent
Linear hepatitis E
Non-segmented
Picornavirus ss(+) RNA No polymerase Naked icosahedral cytoplasm Polio,
Linear Enteroviruses
Non-segmented Rhino
Coxsackie
Hepatitis A
Flaviviros ss(+) RNA No polymerase Enveloped icosahedral cytoplasm Yellow fever,
Linear dengue, St.
Non-segmented Louis
encephalitis
Hepatitis C
Togavirus ss(+) RNA No polymerase Enveloped icosahedral cytoplasm Rubella, WEE,
Linear EEE, Venez
Non-segmented encephalitis
Coronavirus ss(+) RNA No polymerase Enveloped Helical cytoplasm Cornoavirus
Linear
Non-segmented
Retrovirus Diploid RNA dep. Enveloped Icosahedral Nucleus HIV, HTLV,
ss(+) RNA DNA or Sarcoma
Linear Polymerase truncated
Non-segmented conical
Mnemonic: Call Pico and Flo To Come Rightaway
Papovaviruses:
• group of DNA viruses that produce benign & malignant tumors
• virus types:
o Papilloma: HPV types 1-58+. Causes human, cat, dog, & rabbit warts
o Polyoma: found in mice which are asymoptomatic. If induced into newborn animals results in malignant tumors
• virus structure:
o icosohedral capsid virion, dsDNA, circular, replicated & assembled in nucleus
o p53 and RB regulates cell growth but papovaviruses binds to them to promote cell growth
o T angitens regulates transcription with the p53 and RB, T antigen is found w/ viral DNA for replication to continue in cell.
• Papovaviruses:
o Papillomaviruses replicate in the squamous epithelium of skin & mucous membranes to produce warts
o Skin warts, anogenital warts (cervical cancer, 16,18 where E1 & E2 become inactivated so that E6 binds to p53 & E7 w/ RB)
§ Koliocytosis: infectious human papillomavirus of epi layer of uterine cervix or external genitals (condylomata acuminata)
§ Hyperkeratosis: inflammation w/ excess growth of prickle cells of skin
• Treatment warts:
o Remove surgically cryotherapy (liq. N2), electric, or chemical means (ex. podophyllin)
o Laryngeal warts: remove surgically but NOT w/ irradiation b/c could induce malignant changes
o Imiquimod + IFN can promote faster healing applied to topically (ex. external genital regions)
o Cidofovir: antiviral nucleoside inhibits viral DNA polyermase

Adenovirus:
• Virus structure:
o dsDNA, naked, icosahedral, linear shaped, fiber (attachment protein + hemagglutinin activity)
o 47 different types, permissive, found in human adenoids & tonsils
• Important misc facts:
o Associated with conjunctivitis, GI tract, and upper respiratory disease (common cold).
o Produce tumors in baby hamsters but NOT in humans.
o Most common infection of tonsils & adenoids in early child life is from adeno types 3 & 7 and is latent.

Parvoviruses
• The ONLY ssDNA, naked, icosahedral, linear, e-nucleated w/ mature RBC
non-defective defective
replicates only in multiplying host cells requires helper virus (Dependo virus) ex. of
helper virus is (Adenovirusand Herpes Virus)
• Disease:
o Erythema infectiosum (fifth disease) - comment: remember a 5 finger slap in the face you get this redness!
o 1 of 5 common childhood exanthems or rashes (looks like slapped cheek (facial rash)
o Chronic hemolytic anemia (ex. sickle cell anemia) leads to aplastic crisis (lytic on RBC)
o Immunodeficient can lead to chronic anemia
• transmitted:
o vertically (cross placenta) results in spontaneous abortion associated w/ HYDROPS FETALIS

Herpesviruses
• dsDNA, enveloped, icosahedral, linear shaped,
• tegument: contains viral proteins & enzymes involved in initial replication
• Lab diagnosis:
o Tsanck smear (scrapings of infection) see Cowdry Type A cells (acidophilic intranuclear inclusion bodies) see syncytia
• Virus replication:
o Virus induces synthesis of viral thymidine kinase & DNA polymerase
o Phosphonoacetic acid inhibits herpesvirus replication by inhibiting viral DNA polyermase
o Viral protein are made in cytoplasm & enters the nucleus where they assemble w/ the DNA into virus particles
o Virus buds from nuclear membrane, pickup lipid bilayer & viral proteins that have been inserted into it.
o Infection can be lytic or lead to latent infection
Latent in nerves - Herpes Type 1 & 2 (HSV-1 & HSV-2)
- Varicella-Zoster:
a) chicken pox: (varicella portion) acute infection (children)
b) shingles: reactivation of virus of chicken pox
Latent in - Epstein Barr Virus (EBV)
leukocytes-WBC a) Burkitt’s lymphoma (tumor in jaw), nasopharyngeal carcinoma
b) infectious mononucleosis

HSV-1 (usually infects above waist) HSV-2 (usually infects below waist)
- Location of latent: Trigeminal dorsal root ganglia - Location of latent: Lumbo-sacral dorsal root ganglia
- Acute herpetic gingivostomatitis: most common - Genital Herpes: lesions develop in genital organs.
infection for Type 1. Gums swollen! Similar to herpes - Neonatal Herpes: transmitted to new borns
labialis. Oral herpes. Self-limiting. - Aseptic meningitis:mild, self-limiting
- Eczema Herpeticum:(Kaposi’s similar to Herp 8) infect
fingers (herpes whitlow)
- Keratoconjunctivitis: infects eye
- Herpes labialis:(cold sores!!) NOT common cold, most
common recurrent disease.
- Encephalitis: residual neurologic defects

• Varicella-Zoster
Chicken pox (multiple crops) Shingles
- occurs in children via mucosa the upper respiratory tract - occurs in the posterior nerve & ganglia (Dorsal RG)
- swelling of epi cells, eosinophilic inclusions found in - reactivation of the chicken pox
nuclei of infected cells & end up attacking nerve cells - triggered by stress, immunocompromise, trauma
- virus replication occurs in the nucleus
- incubation time: 2-3 weeks
- symptoms: fever, fash, vesicles appear
- differs from small pox (1 crop)

• Treatment:
o Acyclovir, vidarabine, idoxiuridine, trifluoridine

• Cytomegalovirus (Herpes 5) Latent in Leukocytes


o Disease:
§ Most infection of infants, caused by intrauterine or early postnatal infection (pass to child)
§ Teratogenic: virus causes severe congenital anomalies in infants (mental retardation)
§ Cytomegalovirus cells found in the epi tissues of liver, lungs, kidneys, lungs, GI, parotid gland, pancreas, etc.
o Virus recovery:
§ Virus can be recovered from the mouth, urine, liver, adenoids, kidneys & peripheral blood leukocytes
§ Diagnosis of virus infection by “owl cells”or “owl eyes” in the urine. “Basophilic” intranuclear inclusion body
§ Also found in Conventional slow virus disease: involves brain infection
o Treatment:
§ Ganciclovir, foscarnet, cidofovir
§ Vaccine is not effective

• Epstein Barr Virus (EBV) – latent in B cells are leukocytes


o Disease:
§ Burkitt’s Lymphoma & Nasopharyngeal Carcinoma
§ Chromosomal translocation breakages are associated
§ Associated with “Kissing Disease” (mono, results in large lymph nodes & spleen)
• Use Paul Bunnel test to detect mono
§ á WBC ex. Downey cells: B cells infected by EBV
§ Herpes Virus 6:
• Lymphotrophic & ubiquitous associated w/ children’s disease
• Childhood exanthema subitum (roseola)
• Associated with multiple sclerosis
§ Herpes Virus 8:
• Associated w/ Kaposi’s sarcoma, (found in Type I) problem w/ immunocompromised

Poxvirus
• Virus structure:
o Largest DNA, enucleated, complexed shape
• Virus replication:
o ONLY DNA replicates in the cytoplasm
o Replicate in enucleated cells
o Virus DNA and proteins are packaged into virus particles occurring in cytoplasm in the Guarnieri bodies

• Small pox:
o 2 variants, enters in the mucous membranes of upper respiratory tract with incubation of 12-16 days.
o Virus multiplies in lymphoid tissues with infection throughout the body. May result in scaring.
o Diagnosis: Incoluated in embyronated eggs
o Treatment: cidofovir (inhibits DNA polyermase) also it is safer w/ less side effects

• Molluscum contagiosum
o An ex. of a pox virus leads to skin infection, benign tumor, it is a nodular-wart-like growth
o Transmitted: direct contact or fomites & spread by STD
o Treatment: curettage (scrape) or liquid N2

Picornaviruses
• Enteroviruses (intestine)
o Polio and coxsackie viruses
• Rhinoviruses (nose): common cold
• Cardiovirus: in rodents
• Virus characteristics:
o (+)ssRNA, naked, icosahedral, smallest RNA virus, nucleic acid is infectious
o Poliovirus: stable in acidic environment pH 3-9. Found in GI tract & in feces
o Rhinoviruses: unstable in acidic environment pH 5-6. Found in oropharynx area
o Replicates at 33C.
• Poliovirus
o Smallest RNA, 3 serotypes (1-3): 1&2, 2&3 cross react, 1 & 3 doesn’t cross react
o Inactivated by UV, drying. 1M MgCl2 thermally stabilizes b/c of the capsid proteins.
o Immunological properties:
§ C antigencity is the empty capsid which lacks the vRNA. NO VIRUS RNA
§ D antigencity: is the native complete virus HAS VIRUS RNA
o Host: man is the ONLY natural host
o Virus replication: adsorption (complete particles) ==> penetration ==> uncoating ==> translation (replication) ==> synthesis of
vRNA ==> maturation ==> release
o Disease: Starts oral route. Multiples in tonsil, lymph nodes, Peyer’s patches in SI. Asymptomatic shedding
Infection Ingested virus
Alimentary phase Oropharyngeal, virus in throat, intestinal mucosa
Lymphatic phase Tonsils, deep cervical lymph nodes, virus in feces, peyer’s patch,
Viremic phase Blood
Neurological phase CNS, extraneural tissue, regional nerve ganglia

o Infection: 90% in oropharnyx region


§ Abortive: most common form. Malaise, fever, drowsiness, headache, nausea, vomiting constipation, & sore throat.
§ Nonparalytic: asceptic meningitis similar to symptoms from abortive with stiffness & pain in back & neck.
§ Paralytic poliomyelitis: flaccid paralysis, uncoordinated, painful spasms on nonparalyzed muscles
§ Factors alters the disease: fatigue, tonsillectomy áincidence, pregnancy, age, steroids
o Lab diagnosis: CSF á in leukocytes
o Treatment: aridone (binds to receptors change confirmation)
o Prevention & Control: Vaccine
Salk Formaline Inactivation Virus Sabin Live Attenuated Virus
- grown in monkey kidney - grown in human diploid cells in culture
- inactivated could induce polio - given orally: IgA
- boosters needed 3-5 years - long term immunity
- develops IgG & IgM - reverts to virulence
- no herd immunity - herd immunity (virus can’t spread, buffer zone)

• Coxsackieviruses (enterovirus)
o Coxsackie A: Found mostly in respiratory region. flaccid paralysis
o Coxsackie B: Found mostly in the body region. spastic paralysis
o Diseases:
§ Nonspecific febrile illness: Can lead to polio-like paralytic disease
§ Herpangia:fever, soar throat, anorexia, vomiting, abdominal pain, vesicles in palate, uvula. Self limiting. Not herpes!
§ Pleurodynia: chest pain (Bornholm disease: Devil’s grip)
§ Hand-Foot-Mouth: oral & pharyngeal ulcers which may spread to arms & legs w/ mild fever
§ Mycoardiopathy: mycocardial disease in adults & children
§ Common cold: upper respiratory infection
§ Diabetes Melitis:abrupt onset of diabetes after infection w/ coxsackie
o Control:
§ Spread by feces, pharyngeal secretions, sewage

• Rhinoviruses
o Virus characteristics:
§ Common cold, at pH < 6-7 (unstable in acidic environment), found in nasopharyngeal cavity, not found in GI tract
§ H&M strains
o Host: man
o Control: Vaccine is not possible b/c too many serotypes!

Coronarviruses
• Associated with common cold & SARS
• Virus structure:
o (+)ssRNA virus, enveloped, naked, helical, lipid containing, 2 glycoproteins at its surface (E1 & E2 (activtes HA)
• Multiplication:
o Different in sized viral mRNAs transcribed from vRNA
o Occurs in cytoplasm: assembly by budding into ER & Golgi apparatus. Released by fusion via exocytosis
o Multiplication is max. at ~32C

• Pathogenesis:
o Aerosol and large droplets
o Coryzas (swelling of mucosal membrane of oropharynx), sneezing, nasal congestion, etc…
o Some can cause gastroenteritis
• Prevention:
o No vaccine or regulation available

Orthomyxoviruses (Myxoviruses)
• associated w/ the influenza
• Virus structure:
o (-)ssRNA, enveloped, helical, segmented, own polyermase, infectious respiratory disease, 3 forms (A,B,C)
o A type: replicate in humans & animals, B & C type: not as virulent
Hemagglutinin (HA) Neuraminidase ( NA) Ribonucleoprotein (RNP)
- glycoprotein binds to cell receptors - removes sialic acid á virus spread in - code for structural & non-structural proteins
- main antigen against neutralizing antibody respiratory tract - capsid, it goes into the nucleus
• Virus replication:
o Virus attach to receptor (HA) enters cell
o â pH change so fuses w/ membrane & release RNP
o Viral polymerase cleaves host mRNA & uses capped primer to transcribe the vRNP.
o mRNA & vRNA made in nucleus
o HA & NA made in RER ==> SER until cell membrane & gets glycosylated
o Virus made by budding from cell
Antigenic shift Antigenic drift
- re-assortment -b/c of high error of RNA polymerase
- can have co-infection
- can be pandemic
- change in nucleic acid of virus
• Complications:
o Associated w/ lung infection (pneumonia)
§ Pneumonia: primary influenza, combined viral & bacterial, secondary bacterial pneumonia,
§ Reye’s syndrome: encephalopathy & fatty liver associated w/ Influenza B
• Associated w/ Guillain-Barre Syndrome: ascending paralysis
§ Otitis media
• Treatment:
Anti-viral
- Amantadine hydrocholoride & rimantadine
- Tamiflu
- Relenza (works against NA
- drug blocks uncoating (blocks replication)
• Vaccine
§ Vaccine is prepared in eggs

Paramyxovruses
• Virus structure:
o (-)ssRNA, enveloped, helical, non-segmented, HN and F proteins (2 spikes), contain RNA polymerase
o F protein (fusion) causes lyses of rbc when mixed w/ virus
• Replication:
o Synthesis of mRNA & proteins occur at the cytoplasm
o HN & F proteins are assembled in the cell plasma membrane
• Virus types:
o parainfluenza types 1-5: Infects humans & non-humans
o mumps: paramyxovirus
o measles: morbillivirus
o respiratory syncticial virus:
• Parainfluenzas:
o Upper respiratory disease: fever, rhinitis, pharyngitis, CROUP syndrome (laryngoracheobraonch) (barking cough … difficult to
breath)
MUMPS
• Pathogenesis:
o Asymptomatic
o Acute onset of parotitis w painful swelling in salivary glands & transmitted in saliva & respiratory secretions
o Forms multinucleated cells (F protein) causes fusion of several cells (syncytia formation)
o May lead to male sterility because can’t expand due to tunica albuginea
o Stensen’s duct: parotitis
Measles (Rubeola)
• Acute highly infectious disease: w/ rash & respiratory symptoms
o 3C (cough, coryza, conjunctivitis)
o Lacks neuraminidase
o Koplik’s spots: vesicles in the mouth. Appear tiny red patches w/ white specks on buccal mucosa
o Measles can impair CMI (cell mediated immunity)
• Complications:
o Subacute sclerosing panenceophalitis (SSPE):
§ latent in individuals, brain cells have nuclear & cytoplasmic includsions of measles ribonucleoprotein, slow virus
o Herpes 6 (multiple sclerosis)
Respiratory Syncytial Virus (RSV)
o Blocks breathing respiratory tract of young children < ½ year old causes bronchitis & pneumonia .produce syncytia.
• MMR is a live attenuated virus vaccine

Reoviruses
• ONLY dsRNA, naked, icosahedral, 10 segments, double capsid
• Disease:
o Rotavirus: affects children w/ diarrhea. Looks like wheel shaped. Can see genetic re-assortment (shift in orthomyxovirus

Toga Virus (cloak)


• (+)ssRNA, enveloped, 3 proteins
• Rubella (German Measles) – Rubivirus
o Rubella virus:
§ Transmitted by (starts in) respiratory tract. Highly infectious. Spread in blood.
§ Symptoms: rash begins on face, low fever, enlarged lymph nodes & spleen.
• Congenital Rubella Syndrome:
o Complications involve cardiovascular, hematologic, neurologic, opthalmlogic, osteologic, & auditory systems.
o Mental retardation may occur
o Teratogenic: CONGENITAL ANOMALIES
§ Leads to spontaneous abortion
§ MISC: T.O.R.C.H.
To Toxoplasma
R Rubella (German measle) TogaVirus
C Cytomegalovirus & Coxsackievirus
H Herpes Virus

Rhabdovirus
• Virus structure:
o bullet shaped, enveloped, made up of 5 proteins w/ 1-protein outside (glycoprotein G)
o (-)ssRNA, helical, contains RNA polymerase
o Cytoplasmic replication
• Rabies Disease
o Infection of CNS of all warm blooded animals (mammals)
o Transmitted via bite wounds with saliva
§ Rabies multiplies in muscle & connective tissue at the infection.
o 3 stages in human rabies:
§ Prodromal phase: malaise, anorexia, headache, nausea, vomiting, sore throat, & fever.
§ Excitement phase: á salivation & perspiration. Hydrophobia (fear of water) b/c fear of swallowing due to pain
§ Depressive (paralytic) phase: convulsive seizures, coma, and death
• Lab Diagnosis
o Presence of Negir bodies in nerve cells.
o Antibodies can be detected by immunofluorescence, complement fixation, or neutralization.
• Immunity & Prevention
o Passive and active immunization at the same time!
o Give promptly to build antibodies to prevent virus attacking nervous system.
o Vaccines include: human diploid cell vaccine, duck embryo vaccine, nerve tissue vaccine (brains of animals) inject into individuals.

Arboviruses (Flavi, alphaviruses)


• Virus characteristics:
o (+)ssRNA, lipid envelop (less stable), icosahedral
o transferred by arthropods (West Nile virus)
• Disease characteristics:
o West Nile Virus (Flavivirus) spread by mosquitoes. Elder & immunocompromised more at risk for developing encephalitis.
Asymptomatic
o Fevers, encephalitis (fatal, multiplies in non-neural tissue, found in blood,), hemorrhagic fever

Slow viruses
• Incubation periods are long & may appear many years later and are categorized conventional & unconventional.
o Causes: spongiform
o Symptoms include: loss of muscle control, shivering, tremors, & dementia
o There is no inflammation, no immune response, no antigencity
• Prions: Infectious protein, no nucleic acid, cause degenerative neurological disease (scrapie)
o 2 types of prions:
§ PrPc: wildform, native, found on surface
§ PrPsc: infectious form, tertiary structure
o Replication: If PrPsc interacts with PrPc in the body will interact and change confirmation and infect near nerve cell & cause
spongiform appearance.

Conventional Slow Virus Disease Unconventional Slow Virus Diseases


- Subacute sclerosing panencephalitis (SSPE) – measles virus - Kuru: Neuro. disease in ♀ & children. Cannibalism. A prion
- Creutzfeldt-Jakob: dementia, lesions resemble kuru
- Scrapie: agent in brain. Sheep affected. spongiform encephal.
- Transmissible mink encephalopathy: infects brain of mink
- Bovine Encephalopathy (MAD COWS disease): spongiform

Hepatitis Viruses
• Definition: acute infections of the liver (jaundice) may lead to liver cancer (necrosis of hepatocytes)
• Viruses: herpes simplex, herpes-zoster virus, Epstein-Barr virus, coxsackieviruses
o Viruses replicate primarily in the liver (viral hepatits):
§ Hep A (HAV): infectious hepatitis w/ short incubation
§ Hep B (HBV): serum hepatitis w/ long incubation due to blood/fluid exchange
§ Hep C (HCV): non-A and non-B
§ Hep D: super infection, more severe than other Hep, found mostly in pregnant women

• HAV: Picornovirus
o (+)ssRNA, no envelop, enterovirus, spread by fecal-oral route (water, shellfish(raw clams)), infect liver, no cross react w/ Hep B
o Disease: acute infection
• HBV: Hepadnavirus
o dsDNA, enveloped, icosahedral, circular, variable in length.
o Made up of 2 major proteins and 1 minor protein.
o Can be seen as: Dane particle (complete virus), Sphere, or Filament with surface (s antigen)
o Risk in drug abusers, transfusions, high promiscuous populations, infected blood, breast milk, saliva, nasopharyngeal, semen,
menstrual fluid, blood.
o Disease: acute & chronic infection
o Replication: vRNA (pregenome RNA) copied into DNA by reverse transcriptase. Enzyme removes original vRNA and a double
stranded DNA is formed. Virus is formed by budding.
• HCV: Flavivirus
o (+)ssRNA, enveloped, icosahedral, lipid virus, related to plant virus, may cross a mammalian & plant virus
o called non-A & non-B hepatitis
o Disease: acute & chronic infection
• HDV: delta agent (viroid like) Fulminant hepatitis
o ssRNA contains HBV surface protein. HDV is defective virus needs a helper fxn ex w/ HBV
o super infection, intensify and becomes severe mostly in pregnant women

Oncogenes Virus
• RNA or DNA the cancer originates from a single cell
• Function of oncogene: protein kinases codes for tyrosine kinase that phosphorlyates proteins at tyrosine & causes fibrin network
• Human Leukemia-Sarcoma Virus:
o Human Trophic cell leukemia virus HTLV-I and HTLV-II
§ They are diploid cells, w/ 2 copies of genome, NOT segmented
§ HTLV-1 via tax (transcriptional activation) activates IL-1, IL-2 receptor
o HIV
§ Binds to CD4 and chemokine receptor
§ Treatment :
§ AZT (azido-dideoxythymidine)
• Used for viral reverse transcriptase to prevent DNA replication, competes thymidine & terminates DNA
growth. âWBC
• Chain termination which does not allow anything to attach to the genome
§ Anti-protease drugs: (ex. Saquinavir or Ritavir)
• Inhibit protease from cleaving the polyprotein into the virus.
• Protease cleaves the gag & pol to produce nucleocapsid proteins inhibits production of infection but does NOT
cure infection.
• Replication:
o Virus enters, cytoplasm is where the reverse transcriptase copies vRNA into complementary DNA
o DNA is called PROVIRUS & is found in linear & dsDNA form.
o DNA provirus gets into nucleus w/ pregenome chromosome, transcribes w/ infected virus
o mRNA translated into polyprotein (pp) ==> pp cleaved by proteus & assembled in plasma membrane
o virus buds from cell

Glycoprotein (glycosylated)
• gag = group specific antigen, capsid proteins
• pol = polymeras, protease, integrase
• env = enveloped glycoproteins
• LTR = aka long term repeats, these are promoters to enhance transcription factor binding sites

Misc:
What replicates in the cytoplasm? Which viruses do you see coryza?
• pox virus • Coronavirus
• cornovirus • paramyxovruses - Measles (Rubeola) which includes the 3C’s
• paramyxovirus
• rhabdovirus
What is the treatment for polio?
Guillan Baire Syndrome found in? • aridone
• EBV – herpes
• Influenza (ortho)

What is the difference between (+/-) sense?


• (-)sense means infected

What type of vaccine do you need for rabies?


• both live & dead vaccine

What are the 3 phases of Rhabdovirus?


• Prodromal ==> excitement ==> depression

Which are associated with the common cold?


• coxsackieviruses
• rhinoviruses
• adenoviruses
• cornoviruses

Which deals with cold sore?


* Herpes labialis
Parasitology
• definition: reciprocal association, species depend upon another for its existence. Maybe temp. or permanent
• association:
o symbiosis:
o mutualism:
o commensalisms:
o parasite: weaker organism that obtains food & shelter from another & derives all benefits from association
§ Types:
• ectoparasite: live outside of organism
• endoparasite: live inside of organism
• facultative: can have free form or w/in the organism
• obligate: completely establish only inside, can’t live outside
• incidental: establish doesn’t belong (it’s like an accident)
• temporary: part of life cycle in organism, later goes outside
• permanent: remains inside host
• pathogenic: causes injury, by trauma, toxins or damage
• pseudoparasite: like a parasite artifact
• coprozoic: termites able to digest cellulose (in the gut)
§ Needs:
• Moisture & reasonable temperature
• Sources of infection:
o Contamination, food, insects, animals, another person/fomite
• Hosts:
o Definitive host: parasite reaches the sexual stage!
o Intermediate host:
o Paratenic host: it is a transfer host, of a parasite that is not essential to (neither hindering nor hastening) parasite’s life cycle
o Incidental host: accidental
o Dead-end host:
o Reservoir host:
• Parasites can exist in 3 forms
o Sporozite: resistant, quiescent (nothing happening)
o Trophozoite: active eating stage
o Larval forms:worms, cysts
• Lab diagnosis: Intestinal & biliary parasites
o Feces (check 1st), intestinal material, Entero-Test method (get capsule w/ string – parasite wrapped around when taken out)
• Common Human Infections
o Protozaons
o Nemathelminthes (round worms)
o Platyhelminthes (flukes)
o Arthropoda (fleas & ticks)
Protozoan Infections
Disease name/agent Clinical condition Infective form Diagnostic Comments Treatment
Amebiasis intestinal diarrhea, cyst, transmitted via Can go into liver to - metronidazole (M) +
Entameoba histolytica dysentery food, water, anal cause abscesses iodoquinol (intestinal)
intercourse - (M) and dehydroemetine
+ chloroquine
(extraintestinal)
Acanthamebiasis - keratitis (eye) may lead to - trophosoites, eye Topical miconazole for eye
agent Acnthamoeba blindness (contact lenses, soil, infection
sp - Chronic granulomatous water sewage)
amebic encephalitis found in brain tissue
- maybe free living
ameba
Giadrdiasis / Giardia protracted diarrhea and cysts, persons to stool, duodenal animal reservoir - quinacrine HCL,
lamblia malabsorption syndrome person contact, food, contents, cysts, (beavers, muskrats) - metronidazole
water (campers) trophozoites - furazolidine
Trichomoniassis vaginitis, itching, trophozoite vaginal pap smears, males - metranidazole
Vaginal inflammation, discharge transmitted by discharge, (asymptomatic), no cyst
veneral scrapings, form
trophozoite
African - skin lesion at bite site, metratrypanosomes, blood, CSF, Winterbottom’s sign - Gambian,
Trypanosomiasis fever, lymphadenopathy transmitted by bite lymph node (posterior cervical - eflonithine,
sleeping sickness, - CNS involvement: of TseTse fly aspirates, chain enlargement) - Rhodesian,
trypanosome brucei progressive mental, coma, trypanosomes - Suramin,
gambiense, death due to pneumonia - Rhodesian encephalitis,
trypanosome brucei - Starvation, sepsis - Melarsoprol B
rhodesiense

American - Chaga’s disease (in trypomastigoes blood & - xenodiagnosis: - Nifurtimox


Trypanosomiasis / children) acute: visceral (body form) & trypomastigotes person suspected - benznidazole
Trypanosoma cruzi organs, heart, unilateral transmitted by having acute disease
opthalmia, palpebral reduviid bug specific - Host & resvoirs:
edema, eyes (kissing bug) feces antibodies that domestic and wild
- Chronic disease (adults) : bind to T. cruzi animals
megaesophagus, antigens
megacolon, enlarged heart
Leishmaniasis visceral leishmaniasis (kala- Promastigotes & bone marrow - reservoir in dogs & - antimony compounds
azar), destruction of Phlebotomus aspirate & foxes
macrophages, enlarged liver (sandfly) macrophages - thatched roofs,
& spleen, protracted fever breeding places for
sandflies
Balantidiasis diarrhea & dysentery - cysts, transmitted stool, cyst largest protozoal - Iodoquinol
food & water parasite
contaminated by
pigs

Plasmodiasis periodic fever, chills, blood, cyclical drug resistant in Chloroquine PO4,
Malaria, plasmodium hepatosplenomegaly, plasmodial falciparum malaria primaquine PO4, quinine
vivax, ovale, anemia forms in RBC for falciparum malaria
malariea, falciparium
Toxoplasmosis - primary infection ingestion oocyts, serum, tissues, definitive host cat pyramethamine +
Toxoplasma gondii asymptomatic or mild meat containing organism not family sulfadiazine
- congenital infections tissue cysts readily
(anomalies of CNS, eyes) observed or
- immunocompromised: cultured from
disseminated infection humans
involving CNS
Babesiosis / Babesia Nantucket fever, resembles bite of nymph of blood & important infection of clindamycin + quinine
microti malaria hard tick, Ixodes organism in domestic & wild
sacapularis RBC animals & deer mice,
field mice important
reservoir
Cryptosporidiosis profuse, watery diarrhea oocysts, feces of stool, gut Immunocompromised, antibiotics not effective,
animals, human tissue, oocyts fatal diarrhea, immunocompetent self
feces & respiratory observed via dehydration, limiting, supportive care,
secretions phase contrast parenteral nutrition, paromomycin (some
microscopy, supportive success)
acid-fast
stained
material

Helminth Diseases (Round worms)


Disease name/agent Clinical condition Infective form Diagnostic Comments Treatment
Ascariasis/ … Most common in Eggs, feces contaminated Stool/ eggs - Mebendazole
lumbricoides children, abdominal pain, food or soil - pyrantel pamoate
obstruction or worm
migration
Hookworm / necator - Epigastric pain - larvae, (soil, on vegetative Stool /eggs Controlled by sanitary - Mebendazole
americanus or - anemia penetrate skin) disposal of human feces - pyrantel pamoate
ancyclostoma
duodenale
Stronglyoidiasis / Watery, mucous diarrhea Larvae / larvae in soil or Stool /eggs ONLY nematode worm - Thiobendazole
stercoralis vegetation penetrate skin reproduce in host. Free
living & parasitic phase
Enterobiasis / E. Anal pruritus Eggs / ingestion of eggs anal contact Pinworm, seatworm - Mebendazole
vermicularis spec/eggs infection - pyrantel pamoate
Trichinelliasis / Muscle biopsy, serum, curled up Muscle worm infection - steroids (severe)
larvae ingestion of larvae in - Thiobendazole
meat muscle, (adult)
antibodies
Schistosomiasis / Fever, lymph node & Larvae (cercariae), larvae in
mansoni, liver enlargement, snail infested water
haematobium obstruction of vessels of penetrate skin
urinary bladder, intestine,
liver
* Taeniasis / saginata Tapeworm infection, Larvae (cysticercus), Stool / T. solium may cause serious - prazquantel
(beef) solium (pork) adult worm infect ingestion of eggs, worm CNS infection (cysticercosis) (unhooks)
(asymptomatic) beef, raw/undercooked pork or segments
epigastric fullness, nausea beef
* Diphyllobothriasis / Fish tape worm, Larvae (plerocercoid), Stool / - prazquantel
latum asymptomatic, diarrhea, ingestion of raw eggs, worm (unhooks)
anemia b/c lack of B12 fish/undercooked fish segments
* Tapeworm disease / Mild symptoms, loss of Larvae, ingestion of Eggs in - prazquantel
Rat tapeworm / worm infected cereals (flour) feces (unhooks)
Hymenolepsis
diminuta
* Dwarf tapeworm / Abdominal pain, nausea Larvae, hand to mouth Eggs in Most common tape worm in - prazquantel
Hymenolepsis nana & vomiting in heavy inf contact, ingestion of mouse feces southeast US, children (unhooks)
feces affected
8/19/2006

Additional materials
- Levine’s last lecture on parasitology
- Enterococcus
- M. Marium (Group I)
- IMVIC (relevant lactose & gas), pg 33
- UPEC
- Bortonella Quintana
- Coxsacckie
- Coronavirus
- Orthomyxovirus
- West Nile Virus (arbo virus) ==> What family does it belong to? Flavivirus
- Oncogene

What is the difference between polio vs. poxvirus?


Polio (+sRNA) Poxvirus (dsDNA)
- Subgroup of the piconovirus - largest
- vaccinated w/ Salk & Sabin - eradicated

What is the difference btw the measles (rubeola) vs. German measles (rubella)?
measles (rubeola) German measles (rubella)
- acute infectious disease w/ rash & respiratory - subcategory of toga virus
- 3C - (+)ssRNA, enveloped
- no neuraminidase - rash on face
- Koplik’s spot - congenital anomalies
- impair’s CMI
- SSPE
- RSV: blocks resp. tract in young children

You might also like